Jump to content
 







Main menu
   


Navigation  



Main page
Contents
Current events
Random article
About Wikipedia
Contact us
Donate
 




Contribute  



Help
Learn to edit
Community portal
Recent changes
Upload file
 








Search  

































Create account

Log in
 









Create account
 Log in
 




Pages for logged out editors learn more  



Contributions
Talk
 



















Contents

   



(Top)
 


1 November 22  



1.1  Meissner Effect  
3 comments  




1.2  looking for full moons  
3 comments  




1.3  Relativity of simultaneity help in the following case  
6 comments  




1.4  Question about gull behaviour (could apply to any reasonably intelligent animal, really)  
4 comments  






2 November 23  



2.1  Is there a dry lips gene?  
8 comments  




2.2  acidity of dimedone, barbituric acid and acetylacetone  
9 comments  




2.3  How does dietary fiber affect absorption of nutrients/pharmacueticals  
2 comments  




2.4  North Africa from the sky  
4 comments  




2.5  Matter  
9 comments  




2.6  which is the acidic amide in uric acid?  
6 comments  




2.7  muscle metabolism  
2 comments  




2.8  How to flatten the pages of damp books?  
4 comments  




2.9  Brightest stars in the sky?  
19 comments  




2.10  Terrestrial Leeches - Are they harmful to pets?  
3 comments  




2.11  Appearance of the moon 2 million years ago  
5 comments  




2.12  Edman degradation help!  
2 comments  




2.13  acetic anhydride isn't symmetrical?  
5 comments  






3 November 24  



3.1  Why are carbon atoms in carbon nanotubes bonded to only 3 other atoms?  
6 comments  




3.2  Blood glucose levels and exercise  
8 comments  




3.3  cancer  
3 comments  




3.4  Diving support vessel  
2 comments  




3.5  Human Anatomy  
13 comments  




3.6  mind altering drugs  
15 comments  




3.7  rise in temperature  
7 comments  




3.8  Flat-nose feature of the actor in Twilight  
1 comment  




3.9  beetleguese supernova  
3 comments  




3.10  Eye color changing mammal  
4 comments  




3.11  Max. temperature  
10 comments  




3.12  Shape of a black hole  
6 comments  




3.13  How does the Transit of Venus allow calculation of the distance from the earth to the sun?  
6 comments  




3.14  Speed Of Light  
12 comments  






4 November 25  



4.1  Monstrous shape-shifting owl  
8 comments  




4.2  Early Hominina finding in Americas  
9 comments  




4.3  Hood Canal "tide computer"  
8 comments  




4.4  Type of truss bridge  
12 comments  




4.5  Dehumidifiers and climate change control  
6 comments  




4.6  do blue tulips occur in nature?  
5 comments  




4.7  North Sea gas  
13 comments  




4.8  overies  
2 comments  




4.9  Balancing chemical equations  
4 comments  




4.10  How does an increased heart-rate stimulate the body's metabolism?  
4 comments  




4.11  Freezing contents of deeply chilled liquid.  
3 comments  




4.12  Finding power factor  
8 comments  




4.13  Ionic food preserver  
6 comments  






5 November 26  



5.1  age of atoms  
26 comments  




5.2  Both analog and digital possible?  
1 comment  




5.3  Sun looks different in size  
9 comments  




5.4  How big can a Thanksgiving turkey be?  
8 comments  




5.5  hoax? Bulgarian scientist claims alien contact  
12 comments  




5.6  How does the body sense temperature?  
5 comments  




5.7  harmful aspiration  
3 comments  




5.8  Does blood really thicken in cold weather?  
















Wikipedia:Reference desk/Science






עברית

 

Edit links
 









Project page
Talk
 

















Read
Edit
Add topic
View history
 








Tools
   


Actions  



Read
Edit
Add topic
View history
 




General  



What links here
Related changes
Upload file
Special pages
Permanent link
Page information
Get shortened URL
Download QR code
Wikidata item
 




Print/export  



Download as PDF
Printable version
 




Print/export  



















Appearance
   

 






From Wikipedia, the free encyclopedia
 

< Wikipedia:Reference desk

This is an old revision of this page, as edited by 209.244.187.155 (talk)at02:22, 27 November 2009 (Does blood really thicken in cold weather?: new section). The present address (URL) is a permanent link to this revision, which may differ significantly from the current revision.
(diff)  Previous revision | Latest revision (diff) | Newer revision  (diff)

Welcome to the science section
of the Wikipedia reference desk.
Select a section:
Want a faster answer?

Main page: Help searching Wikipedia

   

How can I get my question answered?



How do I answer a question?

Main page: Wikipedia:Reference desk/Guidelines

See also:



November 22

Meissner Effect

Explain Meissner Effet —Preceding unsigned comment added by 59.164.72.230 (talk) 11:34, 22 November 2009 (UTC)[reply]

Have you read our article on the Meissner effect yet? SpinningSpark 11:51, 22 November 2009 (UTC)[reply]

Removed responses that stem from a breach of our guidelines - see talk page. SteveBaker (talk) 16:24, 22 November 2009 (UTC)[reply]

looking for full moons

I am trying to find out when recently or in the next few years there might be a full moon on the first monday of january, but it seems no moon phase calendars use days of the week. Does anyone know of a better one, or possibly when these particular full moons might have occured.148.197.114.207 (talk) 17:48, 22 November 2009 (UTC)[reply]

The last one was 4th January 1988. 5th January 2015 is the next one, then 6th January 2042. I am using Michelsen's "American Ephemeris" which has days of the week. Note that the local date of the full moon might vary from these dates which are UT, i.e. for London.--Shantavira|feed me 18:14, 22 November 2009 (UTC)[reply]
On timeanddate.com you get a calendar with weekdays coupled with dates for the moon phases, but you'd have to browse some years for rare events of course. EverGreg (talk) 20:14, 22 November 2009 (UTC)[reply]

The question is underspecified -- the date of any event depends on what time zone you are talking about. So, what time zone are you talking about? --Anonymous, 05:44 UTC, November 23, 2009.

Relativity of simultaneity help in the following case

Please click on the following link

http://en.wikipedia.org/wiki/Relativity_of_simultaneity Section: The train-and-platform thought experiment

Let the train shown in above link (picture) is moving from north to south and the figure represents the front elevation for observer standing on the platform.

There are two other observers A and B in middle of track such that A is in front of the train and B is on back of the train.

For A, front of the train is front elevation while for B back of the train is front elevation.

For observer standing on a platform, A is on his south; B is on his north side while train is moving from north to south in between A and B.

Now a flash of light is given off East and West (instead of north and south) at the center of the train.OR

If the two bolts of lightning strike the longitudinal sides (instead of front and rear) of the Einstein’s train in thought experiment.

How the relativity of simultaneity be expressed (for A or B and onboard observers) in aforementioned case? —Preceding unsigned comment added by 68.147.38.24 (talk) 19:28, 22 November 2009 (UTC)[reply]

The thought experiment is about when different observers observe something. It's not clear in your version how the east-west flash would be seen by anyone but the guy on the platform. EverGreg (talk) 20:21, 22 November 2009 (UTC)[reply]

Thanks for your reply. Better, draw a diagram on a piece of paper, it'll just take a minute.

My intent is if we perform the Einstein experiment in such a way that a platform guy change his position and stand either in front or back of moving train and if the two bolt of lights strike the longitudinal sides of train instead of its front and rear then would the onboard the train observer and a guy who stand in front or rear of the train, observe the striking of lights at the same time?

OR

In the same moving train (article picture) if the flashes of lights are fired towards its longitudinal sides (instead of towards cockpit and tail) from the light source at the middle of the train then would the observer that is in front or rear of the train and the onboard the train, observe the striking of light at the same time? Because in this case, to me, velocity of train “v “has no role which is added and subtracted (c+v and c-v) in the subsection of the article for platform observer.68.147.38.24 (talk) 00:08, 23 November 2009 (UTC)K[reply]

Ok. If the light ray starts in the centre of the train and goes towards the side of the train, the observer in the back cannot see it. You may imagine that he would move forward and intersect the light ray before it hits the wall of the train, but when you draw this you will find that then the train would then have to be moving faster than the speed of light. EverGreg (talk) 09:31, 23 November 2009 (UTC)[reply]
My intent is if we perform the Einstein experiment in such a way that a platform guy change his position
and stand either in front or back of moving train and if the two bolt of lights strike the longitudinal
sides of train instead of its front and rear then would the onboard the train observer and a guy who stand
in front or rear of the train, observe the striking of lights at the same time? 
Yes. Dauto (talk) 16:08, 23 November 2009 (UTC)[reply]

If an observer at the back of a moving train use a binocular and starts zooming (condition; zooming speed = speed of train) the aforementioned pulses which were given off from the light source at the center, towards longitudinal sides of train, then? 68.147.38.24 (talk) 07:40, 25 November 2009 (UTC) zarmewa k[reply]

Question about gull behaviour (could apply to any reasonably intelligent animal, really)

What's the scientific term used to describe the behaviour that occurs when an animal is feeling conflicted about two courses of action, so instead does a third behaviour, which may be completely nonsensical?

I've been considering this in relation to gulls. Supposing that you know a nesting gull and the nesting gull somewhat trusts you because you feed her regularly. If you approach her nest (say by getting too close to the rooftop she's built in on), her instincts are telling her to swoop down and attack you to drive you away - but she also knows from experience that you're the guy who feeds her regularly, doesn't really want to attack you and possibly gets the urge to fly down and beg for food at your feet (as she has learned that this is how it's done).

So she does neither. Instead, she shifts around uncomfortably, makes odd gabbling noises, alternatively tenses and relaxes her muscles and stares down at her feet until you go away. Yes, this really happens. A gull that doesn't know you will just attack if you don't back off.

Anyone know the name for this? I was thinking displacement, but from reading our article, this term doesn't seem to entirely cover it. --Kurt Shaped Box (talk) 23:36, 22 November 2009 (UTC)[reply]

This seems to have been discussed here: Wikipedia:Reference_desk/Archives/Science/2009_October_3#That_.22deer_caught_in_headlights.22_reaction_-_what_is_it.3F. --Mark PEA (talk) 08:58, 23 November 2009 (UTC)[reply]
(edit conflict) The article Displacement activity seems to deal more specifically with behaviors such as the one you note. This discussion of "sparking over" may also be of interest. Apparently, such behaviors have been especially noted in birds. Further discussions (such as the one beginning at the bottom of this page) can be found by Googling for "displacement reactions" +birds and similar combinations of terms. Deor (talk) 09:12, 23 November 2009 (UTC)[reply]
I don't know if this would be of interest. Bus stop (talk) 09:19, 23 November 2009 (UTC)[reply]


November 23

Is there a dry lips gene?

If I don't use something like chap stick regularly, my lips quickly begin to look like this http://farm4.static.flickr.com/3506/3965747798_661a9087f7.jpg My daughter is just the same. So is there a gene positively identified for lip dryness? 71.161.45.144 (talk) 00:06, 23 November 2009 (UTC)[reply]

It strikes me as improbable that there would be a specific gene that encoded something as specific as this. However, there are probably genes that control the amount of saliva in your mouth and the amount of oil on your skin (which would seem to conflict when it comes to keeping your lips clear). On the other hand, what you eat and how your care for your body are probably just as likely to play a part and that would be culturally determined, rather than genetically. Just out of curiosity - are you a female? In my experience, women seem to have a rougher time with their lips, if you'll pardon the pun. Matt Deres (talk) 04:03, 23 November 2009 (UTC)[reply]
Different IP, same OP. Actually, I'm male. You're probably right about there not being a specific gene, although I remember back in high school when we were learning about genes we did a test to see who had the gene that made phenylthiocarbamide (PTC) taste bitter. That's a strange thing that is specifically encoded( the variation in the shape of the taste bud, that is ). 20.137.18.50 (talk) 12:38, 23 November 2009 (UTC)[reply]
It's only strange to someone who thinks of biology in a very general sense -- someone who appreciates the finer points of biology recognizes that genetic chapped lip susceptibility is an odd thing, while the presence or absence of certain enzymes or receptors is sort of exactly what biologic variability is all about. I mean, you live in the same region as your daughter -- so you are probably exposed to the same elements: excessive cold + wind, etc. You and your daughter may share a similar skin condition (not in the scientific sense, but in the general sense) such that it is a wider, skin sensitivity to, let's say, excessive cold + wind. If you two don't wear gloves, do you both develop red + tender knuckles? DRosenbach (Talk | Contribs) 13:05, 23 November 2009 (UTC)[reply]
I see how it could be our position on the spectrum of sensitivity as opposed to an on/off gene. I wouldn't assume yet that it's a cold sensitivity since she's 3 and stays inside mostly now and still gets it. Maybe I'll dig the humidifier out of the attic and test the theory that our moisture sensitivity lies outside that of our environment. 20.137.18.50 (talk) 13:30, 23 November 2009 (UTC)[reply]
The following is OR but I'm convinced that it is true: lip-lube is literally addictive. If you regularly use it when you don't need it, the natural moisture of your lips decreases, with the results that you see. I personally never use it unless I can actually feel my lips start to get crunchy. Looie496 (talk) 16:26, 23 November 2009 (UTC)[reply]
I was going to crack a joke about that comment, but as it stands, I think you're probably right and it's part of the same sort of regulatory feedback that affects our hair (see question on shampooing) and skin. Lip balm is just the tip of it (heh) - the skin cream industry is absolutely immense; I wonder what all those millions of Jergens users would have done a thousand years ago? Cracked and crumpled into dust like that dude in Indiana Jones and the Last Crusade? The more I learn about biology, the more convinced I am that our bodies are smarter than our brains. :-/ Matt Deres (talk) 17:25, 23 November 2009 (UTC)[reply]
Although in my experience, using lip balm/chapstick can be part of breaking the pattern that often leads to dry lips. If I lick my lips a lot (particularly when going outside in the cold and wind), that can lead to dry and cracked lips: I assume because I'm licking the oils off. Applying an oily layer every time I feel the need to lick gets me past the stage when my lips are dry and sore (so I don't feel like I need to lick them to sooth them). Once they're comfortable, I can start cutting back on the balm while continuing to not lick my lips. I very rarely use it any more, and I think the regulation must have sorted itself out. 86.140.144.63 (talk) 19:38, 23 November 2009 (UTC)[reply]

Is aromaticity actually a major reason for why barbituric acid is so acidic (pka around 4)? Because at first I thought it was rather acidic for a dicarbonyl (acetylacetone has pka of around 9) but it appears dimedone has a pka of 5.23 ... which is really surprising. Maybe the "aliphatic ring" factor of cyclic diketones has much more to do with it (the fact that the bonds cannot rotate and thus the carbonyls are more likely to be in the same plane?). In general, does aligning two sp2 centers (initially disconnected) on the same plane actually provide much of the impetus for those "exceptional" C-H bond dynamics more than aromatic stabilisation does? Is it the fact that the ring is heterocylic that explains why the pKa difference bewteen dimedone and barbituric acid is only 1.22? John Riemann Soong (talk) 01:59, 23 November 2009 (UTC)[reply]

The article for Baribituric Acid you link above actually explains the acidity of the acid. If you have reason to doubt that explanation, then a source which refutes it directly may be helpful. --Jayron32 04:05, 23 November 2009 (UTC)[reply]
I actually just wrote the explanation for that article .... however, I said it was both aromaticity and the dicarbonyl effect without saying what was the relative contribution of both. That's because I'm unsure what is the more significant factor. John Riemann Soong (talk) 04:32, 23 November 2009 (UTC)[reply]
Well, what you would need is a non-aromatic but similar structured compound, and/or a one without the dicarbonyl, but otherwise similar. Then you could get a very rough estimate as to the contribution of each part; but it is pretty impossible to completely isolate each effect. The two effects likely work together in a way to reinforce each other such that each in isolation would not sum up to the total effect. --Jayron32 04:35, 23 November 2009 (UTC)[reply]
Actually what I note is that dimedone is already fairly acidic, even though its conjugate base is not aromatic! Is it because aliphatic rings already have some aromatic character? Do the two methyl groups cause steric hindrance that makes the ring more planar and less chairlike? The surprising thing is that the two effects don't seem to be synergistic. The aromatic stabilisation appears to only give a pKa drop of around 1.23. Why is this so? Is it antagonistic cross-conjugation effects? John Riemann Soong (talk) 04:41, 23 November 2009 (UTC)[reply]
Interesting -- actually the loss of the proton could arguably give rise to antiaromatic system ... do I have the right acidic proton? :S (It's not one of the amide protons?) What would be "nature's way" of remedying this -- simply have one of the carbonyl pi electrons not participate in the system? Perhaps the threat of antiaromaticity might actually stabilise an enamine bond (on top of an enol one)? John Riemann Soong (talk) 04:45, 23 November 2009 (UTC)[reply]
Careful with the "aromatic" diagram...the negative charge is on the oxygen atoms outside the ring, not in the ring itself. Acually each oxygen is –1 and the ring itself is +2: enolate makes O and (neutral) alkene in ring and each amide in its resonance form makes each of those O and N+ and neutral enamine gives 3 π in ring. That's 6 e...what what enamine (leading to antiaromaticity) are you seeing? Regarding which H is lost, phthalimide has pKa=8.30 (measured in water), which is the most acidic simple imide I can find and more acidic than almost any doubly-enolizeable αH I see. Except acetylacetone with pKa=9 (measured in water). Not sure what conclusions to make, except that WP:V forbids you from making your own analysis of the situation in the article. DMacks (talk) 05:18, 23 November 2009 (UTC)[reply]
The interesting thing is that the enolate / carbanion / enamine / amide forms are in resonance, but some resonance structures are possibly antiaromatic, while some are aromatic. IIRC the electron density is concentrated in both the centre of the ring and on the carbonyls (I put dashed lines..., i.e. O is only d-) I might upload a Hartree-Fock calculation at some point. When are carbonyl electrons part of a ring, and when aren't they? The sp2 carbon pi orbital in a carbonyl is usable by an aromatic system, but the pi electrons of that carbonyl don't actually participate? John Riemann Soong (talk) 05:49, 23 November 2009 (UTC)[reply]
I'm sorry, I have no idea what you are talking about with the "carbonyl electronic part of a ring", etc. If you do normal resonance of carbonyl, the electrons move out to the O, not in to the C. If the C but not the O is in the ring, the normal resonance makes the electrons not part of the ring. I can't think of any reasonable resonance except the『α carbanion』form that has negative on the ring itself, and there are several resonances that put N+ on the ring itself. But more importantly, if you are considering aromaticity, the only way you get a 4n+2 electron count is by having a net +2 on the 6 ring atoms and –1 on each oxygen. All six atoms are not the same in a charge-distribution sense, but the diagram is "net for the whole ring". The logical extension of your idea as you are explaining it is that α deprotonation at C5 of 2,4-cyclohexadienone would be "δ on O but mostly –1 on the ring". But I don't think anyone considers that a reasonably good approximation of phenoxide. DMacks (talk) 06:22, 23 November 2009 (UTC)[reply]

How does dietary fiber affect absorption of nutrients/pharmacueticals

I was curious of the mechanism of why fiber negatively affects the absorption of some (but not other) nutrients and medications. --68.103.143.23 (talk) 02:47, 23 November 2009 (UTC)[reply]

I know that it can speed up the digestive cycle. I'm afraid I don't know more. 66.65.141.221 (talk) 19:23, 24 November 2009 (UTC)[reply]

North Africa from the sky

We've been looking at aerial pictures of North Africa and have noticed that, while Egypt and Tunisia are nearly entirely sand-coloured and white, its immediate neighbours, especially Libya but also North Niger, Chad and parts of Algeria, are a magnificent array of colours, including silver and a varied pallete of blues and browns. Can anyone explain what accounts for the difference?

All the best

--77.211.105.58 (talk) 14:16, 23 November 2009 (UTC)[reply]

That may depend on your source of aerial imagery. In NASA's famous 'Blue Marble' satellite image series, very nearly all of North Africa is uniformly covered by the Sahara desert, as seen in this image. On the other hand, if you look at the 'satellite' view (actually a combination of satellite and aerial images) in Google Maps, you'll see significant changes in the appearance of the Earth as you move westward from Egypt, through Libya, into Tunisia. Obviously the sands of the Sahara desert don't abruptly change colour at the Egyptian border. Instead, what has happened is Google has licensed aerial and satellite imagery from a number of different sources; the Libyan were collected and manipulated differently than the Egyptian data. It's a matter of choosing different settings for contrast and colour correction. In the Google Maps images, I'd say that the Egyptian colour choices are probably more 'realistic', but the Libyan data set has more contrast and reveals more detail. It's a matter of aesthetic preference, and also of what further use to which you'd like to put the images. TenOfAllTrades(talk) 14:51, 23 November 2009 (UTC)[reply]
It's actually really hard to get photos of large areas of the world that have comparable coloration. The trouble being that some photos come from satellites with one kind of sensor - others with another - yet others from aircraft flying at different altitudes. All of that results in different amounts of shifting of the color depending on how much air is in the way. Then, it also depends on the sensitivity of the film or digital sensor, the duration of exposure. Also, on what time of day and what time of year the photo was taken - and whether it was cloudy or clear. In remote parts of the world, the amount of time since the last rainfall makes a huge difference. So all in all, it would be pretty remarkable if these photos ever matched up! That said, many photographic sources will attempt to correct for all of these vagiaries - with varying degrees of success! SteveBaker (talk) 00:15, 24 November 2009 (UTC)[reply]

Matter

Is water the only substance found in all three forms: liquid, gas and solid? —Preceding unsigned comment added by Divyam21 (talkcontribs) 16:11, 23 November 2009 (UTC)[reply]

No. Pretty much any element or compound can be found in all three forms. Few other than water can be found in all three forms at everyday temperatures and pressures, though. Room temperature is very close to the triple point of water (which is about 0°C), so we see all 3 quite often. --Tango (talk) 16:37, 23 November 2009 (UTC)[reply]
Tango makes several correct statements but incorrectly joins two of them with "so". In a zero-pressure environment, no matter how close the temperature was to the triple-point temperature, you would never see liquid water. --Anonymous, 22:15 UTC, November 23, 2009.
Tango surely did not mean we often see all 3 forms at the same time so his comment is valid. Most of us are seldom if ever in a zero-pressure environment so how is that relevant? Cuddlyable3 (talk) 19:01, 25 November 2009 (UTC)[reply]
You have missed my point. If we say "A, so B" it means that if A is true, we can always conclude, from that alone, that B is true. In this case A and B are both true but only because there is an additional reason involved, namely that "most of us are seldom in a zero-pressure environment". The word "so" was wrong, and that was all I was saying. --Anonymous, 19:52 UTC, November 25, 2009.
Absolutely not. However, water is one of few materials commonly found in all three states naturally on Earth's surface. — Lomn 16:35, 23 November 2009 (UTC)[reply]
Question answered, but to add some fun. The phrase "all three forms" is vague, or incorrect really. There are a lot of forms of matter. Chris M. (talk) 19:19, 23 November 2009 (UTC)[reply]
Perhaps the OP meant to ask if water is the only substance found in all three forms naturally on Earth. A Quest For Knowledge (talk) 23:52, 23 November 2009 (UTC)[reply]
I"m sure they did, but there are at least four forms found naturally on Earth. Plasma is very common on Earth]. :) Chris M. (talk) 12:32, 24 November 2009 (UTC)[reply]
Sulphur can be found in all three states "naturally" on Earth's surface - although it takes a volcano to turn it into a gas. Of course it takes some pretty hot hot-springs to get water into it's gaseous form - steam isn't usually present in most parts of the world! Sulphur melts at 115 degC and boils at 444 degC. Lava is around 700 to 1400 degC and sulphur is commonly found in volcanic regions. Of course it's possible that it may burn before it boils depending on the conditions. Iodine and bromine are other good candidates - but they aren't exactly commonly found just lying around on the earth's surface. Bromine melts at -7 degC and boils at only 58 degC - so it has an even "better" temperature range than water if you're interested in seeing solid, liquid and gas at close to room temperature. SteveBaker (talk) 00:06, 24 November 2009 (UTC)[reply]
Also note the subtle distinction between gaseous water vapor and water mist, which is liquid water that has condensedbynucleation. Though it can waft and convect, it is properly liquid water that is being buoyantly lifted. Water mist and true gaseous water vapor can coexist in equilibrium, so the distinction is subtle. Nimur (talk) 01:47, 24 November 2009 (UTC)[reply]

which is the acidic amide in uric acid?

Google is being obnoxious. I suspect (from a hint I got somewhere) it's not the amide proton between the two carbonyls. Why not? Help! John Riemann Soong (talk) 16:20, 23 November 2009 (UTC)[reply]

Can someone also quickly do an HF-3-21 charge density calculation for me? John Riemann Soong (talk) 16:21, 23 November 2009 (UTC)[reply]

Once again, the article on Uric Acid explains this. The acid is diprotic; the more acidic hydrogen is the one on the 6-membered ring, and the second hydrogen is on the 5-membered ring. My guess is that the most acidic proton IS the one between the two carbonyls, but if you have reason to suspect why not beyond "gut feeling" you may have sources which explain so... --Jayron32 21:41, 23 November 2009 (UTC)[reply]
Phtalimide's pKa is only 8.3 but uric acid's pka is half that so I suspect it may be a different motif. John Riemann Soong (talk) 22:14, 23 November 2009 (UTC)[reply]
Uric acid has many more oxygens and nitrogens that phthalimide, so i suspect that the inductive effect has a whole lot to do with it. The same explanation can explain why perchloric acid is a much stronger acid than chlorous acid. Remember that pKa is a logarithmic scale, so uric acid is about 15000x stronger than phthalimide, but perchloric acid (pKa = -10) is 100,000,000,000x stronger than chlorous acid (pKa = 2), for the difference of two oxygen atoms. --Jayron32 02:46, 25 November 2009 (UTC)[reply]
I looked at all the crystal structures of urate salts I could find, and the missing proton is always the one on the 6-ring not between carbonyls.
The simplest explanation I could think of for this proton being the most acidic is that the anion formed has more good resonance structures (4) than the anions from deprotonation at two of the other three sites (3). It doesn't explain why the proton on the same side but on the 5-ring is not lost as the corresponding anion also has 4 resonance structures.
I drew some structures to illustrate my point.
Ben (talk) 18:57, 25 November 2009 (UTC)[reply]

muscle metabolism

which metabolic pathway is used in contracting muscles while jogging? —Preceding unsigned comment added by 207.230.204.2 (talk) 19:16, 23 November 2009 (UTC)[reply]

See muscle contraction#Skeletal muscle contractions. Looie496 (talk) 19:53, 23 November 2009 (UTC)[reply]

How to flatten the pages of damp books?

During heavy rain today the ends of two books in my bag got wet, and now the ends of the pages are not straight but wavey. Does anyone have real knowledge and personal experience of how they can be got back into being straight again? I've just read a few pages about how to dry damp books on the internet - can anyone add anything to that? I have a woodworker's vice that could put a lot of pressure on them - but the internet pages say use a moderate weight. I'm wondering if the vice would be better. 84.13.162.136 (talk) 20:14, 23 November 2009 (UTC)[reply]

If you use the vice, ensure that the pressure is even across the book. You really want a book press. I'm not sure why I can't find an article about them on Wikipedia. The main difference between a vice and a press is that the vice just squeezes while the press squeezes with uniform pressure to ensure the binding isn't messed up. -- kainaw 23:00, 23 November 2009 (UTC)[reply]
Be very mindful of the type paper, ink and coating. Some coatings act like glue and if the pages are closed before drying or exposed to high pressure or heat will stick together forever. Some book stores or print shops may have proprietary techniques for book restoration but the process may cost more than a new copy. 71.100.11.112 (talk) 07:23, 24 November 2009 (UTC) [reply]

One of the books is now perfect again, with no sign of water damage. The other book, which had black and white photos on a set of glazed paper pages, has had these pages stick to adjacent pages, as I left it under a lot of weight. With hindsight the best thing to do with the glazed pages would have been to stand the book up with its pages fanned out so that they did not touch until completely dry, and then compress in my vice for a few days to flatten any remaining waveyness. 92.24.40.108 (talk) 12:01, 24 November 2009 (UTC)[reply]

Brightest stars in the sky?

I live in an area with a huge amount of light pollution. At best I can only see about one star in the sky at night. About half or more of my sky is blocked by buildings. I am near London, and looking south. Can anyone have a educated guess as to what the star is I can see? I suppose this is complicated by the possibility of a bright "star" actually being a planet. Sidenote - earlier this evening I was in a darker area, and I was surprised to see my shadow cast by the moon behind me, which was not even a full moon! I've never seen that before. 84.13.162.136 (talk) 20:27, 23 November 2009 (UTC)[reply]

Right now, Jupiter is particularly bright and generally overhead in the early evening. There are also numerous star charts available across the web that could help you narrow down the candidates. — Lomn 20:36, 23 November 2009 (UTC)[reply]
I'd second the Jupiter guess. It's visible during your evenings, and easily the brightest non-Moon thing in the sky right now. If you look closely, does it seem to have a different color? If so, that should confirm it, as would the fact that it probably appears slightly larger than your average star. I live in New York City, so Jupiter is usually the only thing in the sky I can see. As for your side note, cool! Still, it's nothing compared to 400 years ago; back then Jupiter and the Milky Way would cast shadows at night!! ~ Amory (utc) 20:42, 23 November 2009 (UTC)[reply]
Sorry, but I don't buy that last bit. I've been out in some very isolated unelectrified parts of the world, and while the Milky Way is spectacular from such a vantage point, it's not going to cast shadows. The full moon has apparent magnitude of -12.6; Jupiter's maximum apparent magnitude is -2.9. Jupiter is one thousand times dimmer! The Milky Way, as a visible band, is ballpark mag 4, another 100 times dimmer than Jupiter. It's not going to cast a discernable and distinct shadow. There's also the problem of soft light. Even if it's dark enough for starlight to illuminate the ground (and I'll buy that), said starlight is effectively evenly distributed. Jupiter may well be the brightest object in the sky under some conditions, but it's less than 10 times brighter than a great many stars. What meager shadows any one would produce would be washed out by the hundreds of other sources of illumination. — Lomn 22:23, 23 November 2009 (UTC)[reply]
According to NASA ([1]), Jupiter can't cast shadows, but Venus can. According to Bortle Dark-Sky Scale, the brightest parts of the Milky Way can cast shadows under ideal conditions. While the Milky Way is pretty dim it is very big, so that light adds up (see Surface brightness for a discussion about how the light from extended objects is measured). --Tango (talk) 22:37, 23 November 2009 (UTC)[reply]
Lomn: how dark was the area you visited? Did you see the zodiacal light and gegenschein? If the zodiacal light didn't appear annoying bright and you can see clouds as more than holes in the sky, your area probably wasn't as dark as it was 400 years ago. --Bowlhover (talk) 23:23, 23 November 2009 (UTC)[reply]
I think, Lomn, you missed the 400 years ago part of my post. ~ Amory (utc) 04:37, 24 November 2009 (UTC)[reply]

Thanks, although I really meant in general rather than just recently. And in the past I recall seeing a star in the sky even when its still daylight - would that be Venus? 84.13.162.136 (talk) 21:03, 23 November 2009 (UTC)[reply]

Yes, Venus [2] a fortnight ago (and quite regularly). Dbfirs 21:27, 23 November 2009 (UTC)[reply]
Jupiter is right next to the moon right now, so you should be able to identify it. If it isn't Jupiter, then it is probably Venus. --Tango (talk) 21:38, 23 November 2009 (UTC)[reply]
Right around sunrise and sunset (before, during and after), Venus is generally the brightest thing in the sky apart from sun and moon - so if you can see only one "star" - and if it's around that time of day - the odds are very high indeed that you're seeing Venus. Another surprisingly bright thing you see sometimes at around about the same time of day is the International Space Station - but it's quite distinctive because it's moving so fast. SteveBaker (talk) 23:50, 23 November 2009 (UTC)[reply]

What I meant was over a period of years rather than just the last few days or weeks. Unfortunately this has not been answered, so I'm still none the wiser. 92.24.40.108 (talk) 11:22, 24 November 2009 (UTC)[reply]

It isn't possible to answer definitively for "a period of years" because the directions of the stars change regularly over the course of a year while the directions of the planets (which means "wanderers") change less regularly (though still predictably). In your described situation and location, the only star-like objects you are likely to be seeing are the brightest star Sirius, which is visible (at night) low in the south during Autumn and Winter, or the two planets Venus and Jupiter, both of which are generally even brighter than Sirius. For many weeks past, Jupiter has been very prominent in the late evening Southern sky, and is therefore almost certainly what you've been seeing lately. In the past (and in the future) you might have seen (or may see) the sometimes-even-brighter Venus, which however is currently visible only in the East before dawn. There are many books, magazines and online sources that can help you keep track of what is visible in the sky: this one, [3], for example. 87.81.230.195 (talk) 12:18, 24 November 2009 (UTC)[reply]
To be honest I'm getting sick of hearing about Jupiter. OK, I'll take a few years off work, study for a degree in astronomy, and then I'll have the answer. 92.24.40.108 (talk) 12:52, 24 November 2009 (UTC)[reply]
To give you a reasonable answer, we'll need to know a lot more details about WHEN you saw the star, and in what direction. Different stars are visible at different times of the year. I've seen Jupiter, Saturn, Mars, Sirius, and Venus during the evening, and all of them were bright enough and isolated enough to be considered the "brightest star in the sky". --Bowlhover (talk) 16:05, 24 November 2009 (UTC)[reply]
Sirius?Jørgen (talk) 12:07, 24 November 2009 (UTC)[reply]

Perhaps I can re-phrase the question: what are the top ten brightest stars (not planets) in the sky (viewing from London and ignoring those only seen near the horizon)? Thanks. Edit: I've found List of brightest stars but this does not tell me which are visible from London. 92.24.40.108 (talk) 12:44, 24 November 2009 (UTC)[reply]

Most stars will be visible from London at some time of the year. Only those in the far south of the sky will always be below the horizon. I suggest you go to http://www.heavens-above.com, you can say your are in London, enter a time and date, and see what stars are where in the sky at that time from London. Brighter stars are shown as bigger dots and planets are shown in different colours. All interesting things are labelled. --Tango (talk) 13:09, 24 November 2009 (UTC)[reply]

Okay, the latitude of central London is about 51°30'N, so any stars between +90° and -51°30' declination will be in the sky at some time. In practice make that +90° to say -47° because stars very low in the sky are likely to be dimmed by atmospheric effects and blocked by obstacles; it doesn't matter anyway for this purpose because there aren't any very bright stars between -45°0' and -52°0' anyway. Stars between +51°30' (or in practice say +55°) and +90° will be visible all the time at night if it is clear; outside that range, the lower the declination the narrower the time window to see the star.

The following table is based on the table of brightest stars in A Field Guide to the Stars and PlanetsbyDonald H. Menzel (my copy is dated 1964). I have used the forms of the names that Wikipedia does. The table shows the 16 stars that can be brighter than magnitude 1.0; Betelgeuse is slightly variable, so it comes and goes from that list. (I have shown its ranking as the book does, which I presume is based on some average brightness.) Of the 16 stars, 11 are in the range of declinations visible from London at least sometimes, and I have numbered these at left, and shown the declination and magnitude for each star.

   1. Sirius           -16°39' decl. -1.42 mag.
      Canopus          -52°40'       -0.72
      Alpha Centauri   -60°38'       -0.27
   2. Arcturus         +19°27'       -0.06
   3. Vega             +38°44'        0.04
   4. Capella          +45°57'        0.05
   5. Rigel             -8°15'        0.14
   6. Procyon           +5°21'        0.38
      Achernar         -57°29'        0.51
      Beta Centauri    -60° 8'        0.63
   7. Altair            +8°44'        0.77
   8. Betelgeuse        +7°24'      variable, 0.4 to 1.3
   9. Aldebaran        +16°25'        0.86
      Alpha Crucis     -62°49'        0.9
  10. Spica            -10°54'        0.91
  11. Antares          -26°19'        0.92

--Anonymous, 23:10 UTC, November 24, 2009.

Thank you Anonymous for your hard work. 78.146.176.198 (talk) 23:21, 25 November 2009 (UTC)[reply]

It wasn't hard, but you're welcome. --Anon, 04:34, Nov. 26, 2009.

Terrestrial Leeches - Are they harmful to pets?

I never knew about terrestrial leeches before this week but there seem to be quite a few of them in the yard of the house I just moved into!

They're ugly little things, mottled grey in color and shaped like fat, deformed slug-like crescents less than an inch long.

I haven't been able to identify the species yet. My online searches have not been very productive and I am concerned that they might be harmful to pets.

Are these leeches dangerous to cats or dogs?

Is there a resource that can help me to determine the species? We are located close to the shoreline of San Francisco Bay, if that helps to narrow the focus.

Thanks! —Preceding unsigned comment added by 24.6.75.255 (talk) 21:22, 23 November 2009 (UTC)[reply]

The Wikipedia article on Leeches seems to indicate that "land leeches" do not burrow into the skin; and that removal should be easy. this book is listed as the source. IANWEWKATS (i am not whatever expert which knows about this stuff), but that book may be a starting point to look. --Jayron32 21:37, 23 November 2009 (UTC)[reply]
Leeches are not good for cats or dogs, just as they are not good for you either. Does your pet have too much blood? You can remove them by sprinkling with salt, or some other irritating substance. If you just try to pull them off it can be very hard, and the wound bleeds for ages. Graeme Bartlett (talk) 11:16, 24 November 2009 (UTC)[reply]

Appearance of the moon 2 million years ago

KageTora says that he's been reliably informed that at the dawn of humanity, the Moon was so much closer to Earth that it appeared twice as large in the night sky. Is there any truth in this? Neither of us have found anything great on the Internet. We know the moon is moving away at 3.8 cm per year - suggesting it was in the same place as Earth less than ten million years ago (I'm...unsure if my maths is right there). Obviously, the speed of that movement must have changed somewhen, somehow...so simply working out the distance some given time ago with just that number didn't work. Anyone know what we've missed? Vimescarrot (talk)

This site appears to summarize the issue nicely. Also, your back of the envelope calculations appear to have missed the km-to-cm conversion. 380000 km divided by 3.8 cm/yr is 1011 years, not 107. Addressing "double the size": At the dawn of humanity, certainly not. Doubling the size means increasing the apparent radius by 1.4 (the square root of 2). That means moving the moon in by 30% (based on the reciprocal of 1.4). 30% of the 1011 years from the back of the envelope calculation above is still 30 billion years, double the age of the universe. However, at the time of the moon's formation (assuming the giant impact hypothesis), there's a good chance that, briefly, the moon was double the apparent size it is today. Not that anything would have been around on the thoroughly molten surface of Earth to appreciate it. — Lomn 22:10, 23 November 2009 (UTC)[reply]
Actually, 380000 km divided by 3.8 cm/yr is 10^10 years. Moving the Moon by 30%, then, would require 3 billion years, not 30 billion. --Bowlhover (talk) 23:16, 23 November 2009 (UTC)[reply]
Given that the interaction that transfers angular momentum to the moon is gravity, and that gravity is an inverse-square force, it's clear that the forces involved have been much larger in the past, and hence that the process must have slowed down over time. --Stephan Schulz (talk) 22:31, 23 November 2009 (UTC)[reply]
The process that transfers angular momentum is actually tidal friction, which actually follows an inverse cube law. Also, the arrangements of the continents alters the movement of ocean tides, which in turn affects the rate of tidal acceleration. Probably neither of these factors are significant over a 2 million year period, however. --Bowlhover (talk) 23:16, 23 November 2009 (UTC)[reply]
A note as to the title of this thread: the moon formed at 4.5-4.6 billion years ago, about the same time as the Earth.
Switching tacks, the arrangement of the continents, and in particular, the continental shelves, is actually very important as to the recession rate of the moon. The more shallow areas there are that are covered by water, the more friction will result from the tides. So while supercontinents exist, there is much less coastline. In the early Precambrian, higher heat flow resulted in shallower oceans. Awickert (talk) 23:37, 25 November 2009 (UTC)[reply]

Edman degradation help!

I'm stuck because I can't find any further overlaps and I don't know what the "trick" is.

trypsin digestion

Ser-Glu-Phe-Ala-Gly-Leu-Ile-Lys ==
Gln-Ala-Gly-Phe-Pro-Tyr-Ser-Gln-Ile-Ala-Gly-Thr-Lys ++ 
Gln-Glu-Phe-Val-Tyr-Arg OO 
Glu-Asp-Phe-Leu-Ala-Asn-Ala-Gly-Pro-Phe-Arg ==

***
chymotrypsin digestion

Ser-Gln-Ile-Ala-Gly-Thr-Lys-Leu-Ile-Ala  ++ 
Ala-Gly-Leu-Ile-Lys-Glu-Asp-Phe ==
Gly-Lys-Gln-Glu-Phe OO 
Leu-Ala-Asn-Ala-Gly-Pro-Phe  == (internal) 
Arg-Gln-Ala-Gly-Phe ++ 
Arg-Glu-Val-Tyr


(Symbols are where I've found matches).

The amino acid is 46 aa long -- that's my other problem; even without overlapping and randomly joining my sequences together I can only get 45 aa.

I've managed to join them some of them up to yield these fragments:

Quite stuck though. John Riemann Soong (talk) 22:21, 23 November 2009 (UTC)[reply]

Since you're really stuck, then I'll try to get you un-stuck without doing the whole problem. I find it easier to use single-letter codes. I will refer to your tryptic peptides as T1, T2, T3, T4, and your chymotryptic peptides as C1, C2, ...C6. We know that each of the Tn peptides must be preceded by K or R, unless it begins with P or is the N-terminus; similarly, the Cn peptides must be preceded by W, Y, F, L, or M (but your peptides contain no W or M, so it's really down to Y or F not sure why L is not a cleavage site, but it's been awhile since I "used" chymotrypsin). Your peptides become:
(KR)SEFAGLIK
(KR)QAGFPYSQIAGTK
(KR)QEFVYR
(KR)EDFLAQAGPFR
and
(YF)SQIAGTKLIA
(YF)AGLIKEDF
(YF)GKQEF
(YF)LAQAGPF
(YF)RQAGF
(YF)REVY
I hope you can see the utility of this more compact depiction, with the parenthetical indication of the potential preceding residue. Thus, your intermediate solution would look like this:
(KR) SEFAGLIKEDFLAQAGPFR (T1+T4)
(YF) RQAGFPYSQIAGTKLIA (C5+PY+C1, i.e. C5 plus C1 linked by "PY" from T2)
The C1 fragment must be the C terminus, because it ends in A which is not a chymotrypsin cleavage site. This, plus the fact that the T1+T4 fragment ends in a R preceded by F, indicating that there will only be a one-residue overlap there, suggests:
(KR) SEFAGLIKEDFLANAGPFRQAGFPYSQIAGTKxxx (T1+T4+T2)
(YF)    AGLIKEDFLANAGPFRQAGFPYSQIAGTKLIA (C2+C4+C5+PY+C1)
That's 35aa, with "x" indicating a residue missing because a small fragment was lost after cleavage. So, now you have something 35 aa long, which is 11 shy of your goal, and you know you have to build the N terminus. Your building blocks are T3, C3, and C6; neither C3 nor C6 ends in "SEF", so you may need to assume that was lost as well. That should give you enough to finish. -- Scray (talk) 02:36, 24 November 2009 (UTC)[reply]

acetic anhydride isn't symmetrical?

It's really surprising to me -- but one C-O bond length in acetic anhydride is significantly longer than the other C-O bond, when I bring up acetic anhydride with some ab initio calculations. The charge on the carbonyl oxygens aren't even the same. Is it my program, or does this reflect real life? John Riemann Soong (talk) 23:13, 23 November 2009 (UTC)[reply]

What aspect of "real life", how good a set of calculations did you do, and what was your initial geometry for the optimization? DOI:10.1021/jp993131z, which scanned a range of geometries and compared with gas-phase structural data, says there are several low-energy conformers, some of which are not fully planar. I don't have a ref for crystallographic/diffraction structure of the solid. DMacks (talk) 04:08, 24 November 2009 (UTC)[reply]
Well I initially minimised it with a molecular force field optimiser, which gave me an aplanar structure for some reason (for sterics?) I used HF-3-21G, but I didn't think it'd be that far off. John Riemann Soong (talk) 08:28, 24 November 2009 (UTC)[reply]
Again, "minimization" is sensitive to starting geometry...tends to get stuck in local minima near the start rather than making larger changes to get to the global, and the cite I gave notes that there are several local minima with close energy. For example, if you start with the O=C-O-C=O in a "U" shape, you may not ever twist all the way to an elongated conformation, even though that might be better for avoiding O...O dipole-dipole repulsion. And aplanar is not the same as asymmetric (with respect to bond character, charge distribution, etc.) Could have a propeller shape, for example, where one lone pair on the central O is aligned with one carbonyl pi and the other lone pair with the other rather than one O lone pair aligned with both in a fully planar arrangement. DMacks (talk) 17:32, 24 November 2009 (UTC)[reply]
I skimmed the article, and there are a whole host of electronic and steric effects here. The energy minima are non-planar, and some calculations give different C-O and/or different C=O lengths. DMacks (talk) 21:58, 24 November 2009 (UTC)[reply]

November 24

Why are carbon atoms in carbon nanotubes bonded to only 3 other atoms?

Carbons could theoretically have 4 bonds with other carbon atoms, so why must carbon atoms in carbon nanotubes contain only 3 other bonds to other carbon atoms (thus producing the hexagonal shape)? —Preceding unsigned comment added by Bluehark (talkcontribs) 02:08, 24 November 2009 (UTC)[reply]

Every other bond is a double bond. It is strange that neither our article on carbon nanotube nor the one on fullerene show the double bonds in the diagrams. Would be nice if a chemist reading this with the right graphic software would produce something for us. SpinningSpark 02:25, 24 November 2009 (UTC)[reply]
See, this is what I have a question about. Is every other bond a double bond, or do all have an unbonded pair of electrons facing into the tube? —Preceding unsigned comment added by Bluehark (talkcontribs) 02:28, 24 November 2009 (UTC)[reply]
Apparently, they're sp2 hybridized; because the bonds are symmetrical the "extra" bond is decentralized and the carbons are essentially planar (with curvature imposed by the overall structure). There are no unbonded electrons. -- Scray (talk) 03:43, 24 November 2009 (UTC)[reply]
No, every other bond is not a double bond as all the bonds are equivalent. As Scray said, all the bonds are sp2 hybridized bonds. Jkasd 07:42, 24 November 2009 (UTC)[reply]
Carbon doesn't hold lone pairs very well. For one, it'd be a carbanion. The concepts of conjugation, resonance (chemistry) and aromaticity may be useful. The large (exploding) number of possible combinations of resonating pi bonds in a carbon nanotube is an analogue to a metallic bond built on top of a framework of sigma covalent bonds, which gives rise to carbon nanotubes' interesting properties. John Riemann Soong (talk) 08:31, 24 November 2009 (UTC)[reply]

Blood glucose levels and exercise

Diabetes management says: Other approaches include exercise and other lifestyle changes which impact the glucose cycle. It's not a very helpful article. I have personal evidence that suggests exercise has an near-immediate ability to lower blood-glucose levels. First, is my personal experience generally true, where "immediate" means within 1 to 3 hours post-exercise? If it is true, what exactly is happening to cause the drop? Thanks Bielle (talk) 03:51, 24 November 2009 (UTC)[reply]

Well, the obvious guess, from my naive and uncredentialed viewpoint, is that you've just burned it up, and that's why it isn't there anymore. Do you have any reason to look for a more complicated explanation than that? --Trovatore (talk) 03:58, 24 November 2009 (UTC)[reply]
My understanding may be fuzzy, but . . . blood sugar levels above normal can be the result of either insufficient insulin to open the lock-and-key connection that permits the transfer of glucose from blood into body cells or impaired or absent connection points. As far as I know, exercise burns up sugars that are already in the body tissues and not what is in the blood, so, while the body cells may be calling for more glucose, what is it about exercise that "permits" an otherwise impaired system to increase the transfer out of the blood (and thus the lowered blood-glucose number) and into the cells? Bielle (talk) 04:08, 24 November 2009 (UTC)[reply]
Oh, I see. I don't know, then. --Trovatore (talk) 06:46, 24 November 2009 (UTC)[reply]
Type 2 diabetes#Exercise has an explanation but it's not very clear, it has no sources and it seems inconsistent with other articles I checked - someone should look at it. It looks like exercise stimulates a different glucose transporter than insulin. I can't access it, but the third reference at Insulin resistance seems to discuss this. Zain Ebrahim (talk) 10:55, 24 November 2009 (UTC)[reply]
This is the quote from the article Zain linked: Insulin though, which no longer works effectively in those afflicted with type 2 diabetes, causes GLUT1 to be placed into the membrane. Though they have different structures, they both perform the same function of increasing intake of glucose into the cell from the blood serum [citation needed]. Exercise also allows for the uptake of glucose independently of insulin, ie by adrenaline [citation needed]. It does need cites, certainly, as noted. Is there anything in this assertion anyone can either confirm or can tell me, or direct me, to more? Thanks Bielle (talk) 19:09, 24 November 2009 (UTC)[reply]
Here are a few that might be helpful:
  • Not a reliable source, but here's an interesting read from WebMD.
  • This is interesting but pretty technical: PMID 17327455 - I thought the insight about the effect of obesity was interesting, and there is a sizable literature about PGC-1 and glucose metabolism in liver vs muscle - e.g. PMID 19448691.
  • This one gets more directly at the effect of exercise on blood glucose in type 2 diabetes: PMID 12864738.
Hope this helps. -- Scray (talk) 05:18, 25 November 2009 (UTC)[reply]
Thank you, Scray. It will take me a while to work my way through the PMIDs. I hope my medical dictionary is up to the challenge. Bielle (talk) 06:30, 25 November 2009 (UTC)[reply]

cancer

hello, my dad has colon cancer.. he went through an operation 20 days back, 124 cms of his large intestine a removed.. now doctor s started chemeotherapy.. may b 4 lymph node s infected.. wat diet should he have now and wat al care to be taken?? will he get well completely??? pls do reply.. —Preceding unsigned comment added by Shilpa.upadhya (talkcontribs) 05:36, 24 November 2009 (UTC) Template:RD medadvice[reply]

Hi Shilpa! I hope you understand that we have a serious rule here against responding to requests for medical advice. It doesn't mean we're heartless, and I'm sure everyone here wishes the best for your father. --Trovatore (talk) 09:20, 24 November 2009 (UTC)[reply]
Shilpa, I am sorry to hear about your father. The best people to ask about how to care for him are his doctors, and I am sure they will take your concerns seriously. All the best. --TammyMoet (talk) 10:32, 24 November 2009 (UTC)[reply]

Diving support vessel

2 men RCC is an equipent wich is using in diving support vessel.RCC stands for? —Preceding unsigned comment added by Akhilsvnair (talkcontribs) 05:41, 24 November 2009 (UTC)[reply]

Recompression chamber. Deor (talk) 11:09, 24 November 2009 (UTC)[reply]

Human Anatomy

What is the reason for men having nipples? What purpose do they serve? —Preceding unsigned comment added by NirocFX (talkcontribs) 05:46, 24 November 2009 (UTC)[reply]

See Nipple#On male mammals DMacks (talk) 06:14, 24 November 2009 (UTC)[reply]
The basic reason is simplicity.
Although the human body is constantly changing, overwhelmingly the most concentrated period is during fetal development, with a secondary node at puberty. As Nipple#On male mammals mentions, individuals are first formed on a unisex basis. (IMO, the assertion that this is a "female template" is sexist nonsense, based on the notion that without obvious male genitalia, the individual is "merely" female, even without obvious female genitalia.) In any event, at 14 weeks in a second spurt of development, the urogenital systems differentiate into male or female. As these systems must be in place by birth, clearly the physical development must occur within the womb.
On the other hand, functioning nipples are only needed after puberty. There are three sensible schemes for getting this to happen:
(1) Have nipples develop only in females during fetal development,
(2) Have nipples develop ab initio only in females during puberty, or
(3) Have nipples develop on all individuals, but have them "turned on" only in females during puberty.
The first scenario puts extra strain on the development of the fetus. With all the other changes occuring, why add yet another item to the list of things to be accomplished - and accomplished correctly - for something that will not be needed for another dozen years?
While the second scenario delays the action to a more appropriate time, it demands that females grow an entirely new organ, years after everything else is in place.
The third choice is what actually happens. The potential which all mammals have is activated only in females.
The trade-off is simple to understand when we consider the consequences. Males get non-functioning nipples which cause them no harm. Under either of the other plans, something might go wrong and a female would not grow nipples. Thus what actually happens is safer, in terms of the odds of it coming out right, because it's simpler.
B00P (talk) 09:51, 24 November 2009 (UTC)[reply]
What's important to realize is that in addition to the excellent answer above, it must be noted that it isn't this way because it's simple, or because it's safer. Because it implies it was planned out that way. It is this way because out of all the ways it could be done, this way turned out to be most likely to survive and reproduce, therefore it is what exists. While it is technically that way because it's safer, it's not as if it was picked, only tested out. Chris M. (talk) 12:20, 24 November 2009 (UTC)[reply]
Agree with Chris on this one -- see Occam's razor. It's not so much that this is simpler and therefor it happens, but that it happens, and we thus see that it's simpler. It's all nice and good for us to say what is biologically simpler, but until it happens, we don't really know. DRosenbach (Talk | Contribs) 13:11, 24 November 2009 (UTC)[reply]
Also, be sure to realize that neither "simple is better" and "better is what we have" are accurate statements, or at least both of them aren't necessarily true in all cases. I mean, look at the conjoined airway/foodway we have. You could argue that's "simpler" then two, or you could say it's more complex. But it seems almost certain it isn't "better" in any reasonable way, due to how frequently we die from misusing it (choking). Chris M. (talk) 18:02, 24 November 2009 (UTC)[reply]
Until we've directly compared with an alternative, I wouldn't be so sure. Subtle benefits may escape notice when things are running smoothly. For example, the esophagus is a convenient waste bin for our respiratory system; a great deal of nasty stuff from our upper and lower airways is eliminated by the digestive system. Where would that go if they weren't connected? That's just one obvious example; there may be other more subtle benefits (e.g. immunologic). -- Scray (talk) 23:18, 24 November 2009 (UTC)[reply]
Chris M. is quite right; I certainly didn't mean to imply anything else. B00P (talk) 04:06, 25 November 2009 (UTC)[reply]
Think of nipples as more than just a milk transference device. They are also good for detecting subtle things. They're like a couple of navigational sensors on the Enterprise, if you are familiar with Star Trek. Vranak (talk) 16:59, 24 November 2009 (UTC)[reply]
... are you implying that you can navigate with your nipples? Or use them to avoid incoming asteroids?APL (talk) 17:13, 24 November 2009 (UTC)[reply]
Yes, yes I am. Seriously though, you can detect local weather phenomenon with your nipples. They can also be a gateway to intimacy for sex partners. Vranak (talk) 18:07, 24 November 2009 (UTC)[reply]
I can agree with that. I always find my nipples are indispensible for getting through a dense forest at night during a snow-storm. --KageTora - SPQW - (影虎) (talk) 02:25, 25 November 2009 (UTC)[reply]
Oh yeah - it's easy - you go up to the nearest hunter and say "Show me the way out of this nasty forest and I'll show you my tits!" SteveBaker (talk) 04:54, 25 November 2009 (UTC)[reply]

mind altering drugs

What mind altering drugs were used or known in Ancient Civilization such as THC from hemp and hashish? 71.100.11.112 (talk) 06:57, 24 November 2009 (UTC) [reply]

I'd say "see Psychoactive drug#History", but it isn't very useful. How about Entheogen? Some examples of ancient mind altering drugs listed (but not all understood well today) include: Kykeon, Soma, Peyote, and Ayahuasca. Other possibilities include Amanita muscaria, Psilocybe, Atropa belladonna, Nymphaea caerulea, Datura, Opium, others. Pfly (talk) 09:34, 24 November 2009 (UTC)[reply]
I would guess that the oldest one in systematic use is plain old ethanol. But I don't really know that, and of course it does require more preparation than just picking a leaf or berry or something. Can anyone do better than guess? --Trovatore (talk) 09:52, 24 November 2009 (UTC)[reply]
You'd have to define the terms "oldest" and "systematic use" for the question to have a well-defined answer, but I think you're basically right. Sumer is the oldest known civilization, and they had beer. See Ninkasi. Ethanol has a disadvantage of requiring more preparation than just picking a mushroom or a berry, but it has the advantage of being able to be produced from a wide variety of plants, including whatever grain is a dietary staple in whatever region, instead of being tied to a plant species that only grows in certain locations. Red Act (talk) 11:25, 24 November 2009 (UTC)[reply]
Additionally, beer has some properties that other psychoactive drugs do not have—e.g., it can be used as a potable water source as it resists going bad. That's a pretty big advantage in many cultures! --Mr.98 (talk) 21:53, 24 November 2009 (UTC)[reply]
Beer was known to the Ancient Egyptians and Mesopotamians. --TammyMoet (talk) 10:29, 24 November 2009 (UTC)[reply]
As well as cocaine and tobacco, apparently. --KageTora - SPQW - (影虎) (talk) 17:29, 24 November 2009 (UTC)[reply]
Err, that seems very unlikely. That particular source would not count as "reliable" on here (a second-hand account of a caption of a photo in a French magazine—no peer review, no source, no follow-up, after over 10 years). Since tobacco and cocaine are both "new world" products, it would imply some significant cross-oceanic trading long before that is known to have happened. I would file that as "dubious"/"unlikely"/"bullshit"; certainly, a "needs to display some evidence." --Mr.98 (talk) 21:50, 24 November 2009 (UTC)[reply]
Fair enough, Mr98, apologies for the poor link, but I doubt it that Colorado State University thinks it's 'bullshit', as you call it, considering they have written a paper here about whether it's true or not and basically agreed that it is. Again, apologies, it was just the first link I found after looking for something I had been hearing about for years. When I post links in future, I'll be a bit more careful. --KageTora - SPQW - (影虎) (talk) 02:19, 25 November 2009 (UTC)[reply]
No KageTora...bad KageTora - check that article again. (HINT: It's a good idea to actually read the stuff you post as links!) Your Colorado State University paper CLEARLY says at the top left corner (in red ink no less!): "For teaching purposes only, do not review, quote or abstract"...so don't! It's possible that this paper is used in some course in which students are required to debunk a clearly bogus paper or something - who knows? At any rate - it's certainly not authoritative - if anything, it's further evidence that this is bogus. They ask you NOT to use it as a reference. You really should respect that and not associate their good name with a clearly bogus theory when they specifically ask you not to do that! So I'm with Mr.98 - "dubious/unlikely/bullshit"...we still don't have any evidence. The theory that the Egyptians and the pre-Colombians had met (mostly on the grounds that they both built pyramids) - is pseudoscience (or is that "pseudohistory"?) SteveBaker (talk) 04:50, 25 November 2009 (UTC)[reply]
Well, in that case, maybe the Wikipedia article on Egyptian mummies needs to be adjusted, as it was through there that I found that article linked to in my last post. The link in the Wikipedia article leads to the wrong page, but it said 'Colorado State University' and a Google search on that plus cocaine plus Egyptian mummies led me to the page. --KageTora - SPQW - (影虎) (talk) 17:06, 25 November 2009 (UTC)[reply]
Coca leaf has been chewed by people who live near it for a long time, but more as a mild stimulant than a mind-altering stay up all night party drug. --Sean 13:44, 24 November 2009 (UTC)[reply]
It's quite possible that psychoactive drug use predates humanity, although I'm not aware of any actual evidence for this. The book Food of the GodsbyTerrence McKenna -- which, I note, is generally considered pseudoscience -- puts forth the so-called "stoned ape theory of human evolution". In any case, given the omnivorous diet of hunter-gatherers, it's almost impossible that they could have avoided eating psychoactive plants now and then. Looie496 (talk) 18:36, 24 November 2009 (UTC)[reply]
And some animal species have been observed to become drunk after consuming naturally fermented fruit. Googlemeister (talk) 19:22, 24 November 2009 (UTC)[reply]
Cats and catnip too! SteveBaker (talk) 04:35, 25 November 2009 (UTC)[reply]

rise in temperature

To get an idea of just how much energy is contained by a single neutron or in the alternative how much energy is released by the fusion of two hydrogen atoms how many degrees would the temperature of one pound of liquid water rise if a neutron were converted entirely to thermal energy or the fusion of two atoms of hydrogen occurred and the energy released expressed as temperature rise of water? I know this is a dumb question and no its not homework. Thanks in advance. 71.100.11.112 (talk) 18:06, 24 November 2009 (UTC) [reply]

The neutron mass can be found in the linked article to be approximately mM~940 000 000 eV. Using the convertion factor we get mM~0.000 000 000 15 J. That's the same as mM~0.000 000 000 036cal. That means that 1 neutron will lead to a change of 0.000 000 000 036 oC for a mass of 1 gram. 1 pound has about 454 grams so 1 neutron worth of energy would cause a change of 0.000 000 000 000 079 oC on a pound of water Dauto (talk) 19:06, 24 November 2009 (UTC)[reply]
As for the fusion of two protons, according to Proton–proton chain reaction, that reaction yields about 420 000 eV (about 1/2000 the rest energy of the neutron) so the temperature change would be about 0.000 000 000 000 000 035 °C. Rckrone (talk) 20:05, 24 November 2009 (UTC)[reply]
...which is not very useful for visualization purposes. A better way would be to figure out how many neutrons or proton-proton reactions are needed to cause a temperature change of 1 degree. --99.237.234.104 (talk) 23:10, 24 November 2009 (UTC)[reply]
That would be given by the inverse of the number I calculated above Number of neutrons ~ 1/(0.000 000 000 000 079) ~ 12 600 000 000 000. Dauto (talk) 00:30, 25 November 2009 (UTC)[reply]
To get back to measurable(ish) units, that's about 2x10–14 grams of neutrons. DMacks (talk) 02:52, 25 November 2009 (UTC)[reply]
Or to put it another way, one gram of hydrogen atoms fusing would boil an incredible amount of water, something like a billion liters or something... Vespine (talk) 05:45, 25 November 2009 (UTC)[reply]

Flat-nose feature of the actor in Twilight

Does that feature have a technical name? 20.137.18.50 (talk) 20:06, 24 November 2009 (UTC)[reply]

beetleguese supernova

Given what we know about the star Beetleguese (sp?) if it was to go supernova (or rather had gone supernova a long time ago and now the light is just reaching earth), would the result be a celestial object visible during the day, or would it not be bright enough? Googlemeister (talk) 20:11, 24 November 2009 (UTC)[reply]

SN 1054 was visible in the day, and it was ten times further away than Betelgeuse, so I expect yes. Algebraist 20:18, 24 November 2009 (UTC)[reply]
When it goes, it will be bright enough to read a newpaper by its light, there will be no missing it. See Betelgeuse#Fate. SpinningSpark 23:24, 24 November 2009 (UTC)[reply]

Eye color changing mammal

Is there such a mammal that can change the color of its iris, maybe according to mood or food or whatever? --Reticuli88 (talk) 21:13, 24 November 2009 (UTC)[reply]

FWIW eye colour in humans changes, particularly between the age of birth and about two years old. Very few Caucasian babies are born with dark brown eyes, the tendency is to lose blue in favour of gaining green to brown. --BozMo talk 21:38, 24 November 2009 (UTC)[reply]

I mean besides humans - --Reticuli88 (talk) 21:42, 24 November 2009 (UTC)[reply]

FYI kittens eyes also start off as blue and change colour as they mature [4] [5] [6]. I suspect eyes changing colour from birth to maturity is fairly common in mammals actually. I'm not really sure if that's what you're thinking of on the other hand you did say simply 'besides humans' in response to BozMo's post so it seemed it may be relevant `Nil Einne (talk)
Prostaglandin and its analogs such as latanoprost can darken irises in people and animals.[7]. Irises can change colour after death, as shown in this experiment with pig eyes:[8]. As a tangent, pigmented cornea in dogs can mean "dry eye" or Keratoconjunctivitis sicca. Fences&Windows 18:58, 25 November 2009 (UTC)[reply]

Max. temperature

What is the maximum temperature (climate) that a human body can survive in? --AlexSuricata (talk) 21:40, 24 November 2009 (UTC)[reply]

For how long? Googlemeister (talk) 21:43, 24 November 2009 (UTC)[reply]
I think the word "climate" is intended to imply indefinitely long. Red Act (talk) 22:27, 24 November 2009 (UTC)[reply]
I would guess a lightning strike produces the hottest temperatures that a human can survive, but that doesn't really count as climate. Ks0stm (TCG) 22:22, 24 November 2009 (UTC)[reply]

What I mean is, that I read that in some countries the temperature can reach into the 50sº (even up to mid-50sº) - therefore, if I were a God and slowly started to turn the temperature up, what is the temperature at which most people would start to die? --AlexSuricata (talk) 22:35, 24 November 2009 (UTC)[reply]

Are you now changing your question to refer to something like MaxTemp50?DRosenbach (Talk | Contribs) 01:22, 25 November 2009 (UTC)[reply]
There are many other factors to consider. Humidity, for one. And the prevalence of shelter and/or shade - does it still have to be 55 degrees in the shade, in buildings? Vimescarrot (talk) 22:48, 24 November 2009 (UTC)[reply]
Hottest survivable climate is hard to answer...there is Al 'Aziziyah, where the hottest temperature on earth was recorded, however we do not have an article on the town with the hottest climate. There is also the temporary Heat burst. Ks0stm (TCG) 22:51, 24 November 2009 (UTC)[reply]
Humans have been experimentally exposed to very high temperatures for short periods without harm in dry heat. This paper, for instance, went as high as 90°C for ten minutes. I am certain I have read of a military experiment going to an even higher temperature but cannot locate the paper right now. SpinningSpark 23:14, 24 November 2009 (UTC)[reply]
Doesn't answer the question, but see Extremes on Earth#Coldest and hottest inhabited places on Earth. Fences&Windows 18:41, 25 November 2009 (UTC)[reply]

Shape of a black hole

This is something that's never been very clear to me. In terms of the space it takes up in, erm space, is a black hole actually spherical, or is it more like a conical depression/whirlpool with the singularity at the bottom? --84.69.185.215 (talk) 21:51, 24 November 2009 (UTC)[reply]

The conical depression is just a way to visualize the gravity well—it doesn't mean that is actually how it occupies three dimensional space. (No more than the Sun actually exists in a little depression as shown in some visualizations of gravity). That being said, my understanding is that the equations come out spherical, but you can have things like relativistic jets that give it a definitely non-spherical appearance. This has to do with the fact that though it is a sphere, it is rotating along an axis, or something along those lines. Someone who knows more, please feel free to correct me. --Mr.98 (talk) 22:13, 24 November 2009 (UTC)[reply]
As I recall, the black hole itself is an infinitely small point. I may be wrong however, so I'll let other people verify that answer... Ks0stm (TCG) 22:20, 24 November 2009 (UTC)[reply]
Logic would dictate that an infinitely small point with a gravitational pull all around it (i.e. 360°) should be considered to be spherical, or at least, the pull should be. --KageTora - SPQW - (影虎) (talk) 23:36, 24 November 2009 (UTC)[reply]
A non-rotating black hole has a spherical event horizon. Rotating black holes are a little weirder - they also have a spherical event horizon, but in addition to that they have an oblate spheroidal ergosphere. --Tango (talk) 23:44, 24 November 2009 (UTC)[reply]
Strong gravitational fields distort the shape of space. A solid sphere is a subset of 3-dimensional flat Euclidean space, but the space around a black hole is not shaped like 3-dimensional Euclidean space. The problem is that our minds aren't really designed to visualize this distortion. What's often done to illustrate what's happening is that the picture is simplified to 2 space dimensions, so that space is like a big flat sheet. Here we can picture distortions in the sheet caused by gravity since we have an extra dimension in our imagination we aren't using. It's in this representation where a black hole causes that cone shaped sink. If you try to imagine the same sort of departure from flat 3D space you will have a great deal of trouble. Rckrone (talk) 00:06, 25 November 2009 (UTC)[reply]

How does the Transit of Venus allow calculation of the distance from the earth to the sun?

From a diagram on the Transit Of Venus page, I can see how its transit could be used to estimate part of the geometry required to estimate the diameter of the sun, and hence its distance away. But this geometric calculation appears to be subject to knowing the proportional position of Venus between the sun and earth. How was this obtained, in Captain Cook's day? 92.24.170.160 (talk) 22:04, 24 November 2009 (UTC)[reply]

That proportion could be found using Kepler's laws of planetary motion since the orbital periods of Earth and Venus were known. Rckrone (talk) 22:21, 24 November 2009 (UTC)[reply]
The proportional distance is easily calculated from the maximum angle between Venus and the Sun, as viewed from Earth. Looie496 (talk) 00:05, 25 November 2009 (UTC)[reply]

On second thoughts I'm not sure I do understand the transit method - would anyone like to explain it more please, step by step? It seems to me that you would need to know the distance from the earth to Venus to start with - I presume that was not known. 78.146.176.198 (talk) 23:12, 25 November 2009 (UTC)[reply]

The transit would have to be measured from different points on the earth. The exact time that it starts or stops can then give a difference in the angle to Venus as observed by the two observatories, then triangulation can be used to measure the distance to Venus. Graeme Bartlett (talk) 06:26, 26 November 2009 (UTC)[reply]

OK, although provided you could see Venus, then couldn't you do that at any time without requiring a transit? 78.146.237.244 (talk) 09:38, 26 November 2009 (UTC)[reply]

Speed Of Light

If humanity had not developed senses that could detect light, what would Einstein's idea about it being impossible to travel at or over the speed of light have become? This sounds like a hypothetical question, but what I mean is, is there anything stopping us, besides the existence of light? --KageTora - SPQW - (影虎) (talk) 23:31, 24 November 2009 (UTC)[reply]

I think there are a range of answers, one of which might be that humans might have invented sensors that measure light, much as they've invented sensors that measure other things humans cannot sense but which are measured by humans using those sensors. It's the law that stops us, not the light, so although the existance of light has to do with the law, it is not the existence of light which is the barrier. --Tagishsimon (talk) 23:38, 24 November 2009 (UTC)[reply]
The speed of light is also the speed of radio - the speed of microwaves - the speed of heat (when it's radiating). If (for example) we'd still have been able to discover radio - then I think we'd have made radio telescopes and the question might have arisen that radio waves from far distant quasars or whatever always arrived at the same speed. So presuming that we'd still somehow have been able to develop science and technology at all - then I don't see why we couldn't figure out relativity based on other frequencies of electromagnetic radiation. SteveBaker (talk) 00:04, 25 November 2009 (UTC)[reply]
The speed of light appears as a parameter in Maxwell's equations for electromagnetic fields. So if we understood electricity and magnetism, we would know about the speed of light regardless of our senses. Looie496 (talk) 00:10, 25 November 2009 (UTC)[reply]
Indeed, it was the puzzling appearance of c as a constant in those equations that lead Einstein to formulate relativity. Algebraist 00:14, 25 November 2009 (UTC)[reply]

Well, this is what I'm getting at. 'C' may be a constant, but why does it have to be light? Before the 'discovery' of light, in our hypothetical universe where humans don't perceive light directly, what would have been the constant? I suppose radio waves would answer that question, as said above, but radio comes in waves, whereas light behaves both as a particle and as a wave (or has that puzzle been sorted out yet?). In either case, there are particles which have been found to travel faster than the speed of light, or at least appear to. When we sort that question out, will they become the new 'constant'? The question I am asking really, is why is light so special? --KageTora - SPQW - (影虎) (talk) 02:03, 25 November 2009 (UTC)[reply]

To attempt to answer your questions in order: human-like beings smart enough to have discovered relativity will probably have discovered electromagnetic radiation already (whether or not they are able to directly perceive it), and will recognize the maximum speed prescribed by relativity as the speed of electromagnetic rays. If not, then the maximum speed in relativity will just be a new fundamental constant they'll have to think of a name for. Radio and light are essentially the same thing: they're both electromagnetic radiation, just of different wavelength/frequency/energy. Electromagnetic radiation comes in neither waves nor particles; both of these notions are just imperfect models which are sometimes useful in studying them. There are no particles which go faster than light, or which appear to do so. On your last question, you seem to be thinking the wrong way round: the unfortunately named speed of light is first and foremost the maximum speed anything can go at, and secondarily the speed at which light happens to go (when in a perfect vacuum). Light moves at c because it is massless, and all massless things move at the same speed, so light isn't particularly special in this regard. Algebraist 02:20, 25 November 2009 (UTC)[reply]
Likewise, "light" in the strictest sense, that is visible light, is a narrow range of wavelengths between about 400-800 nanometers. There is absolutely nothing at all special about it compared to other parts of the electromagnetic spectrum except that human eyes are "tuned" to detect radiation in that very narrow range. The speed is more properly called "the speed of electromagetic waves"; there is nothing all that special about "light" compared to any other parts of the spectrum, except that humans are predisposed to paying more attention to it due to our special biology. Indeed, the attention we give to that narrow band of wavelengths is all out of proportion with the part of the spectrum that band represents. --Jayron32 02:30, 25 November 2009 (UTC)[reply]
Notably, our OP seems to think that light and radio waves are somehow "different" - but they aren't. Both exhibit the "wave-particle duality" thing - both travel at the same speed and obey the exact same laws. As you go through the colors of the spectrum from blue down through green, yellow, orange and red - you reach infra-red, then millimeter-wave radio, then regular radio waves - it's a continuous thing. The only thing that changes is our eyes' ability to see them and our ability to pick them up with an antenna. SteveBaker (talk) 03:35, 25 November 2009 (UTC)[reply]
In terms of causality, the main thing that distinguishes "light" from other wavelengths of electromagnetic radiation is that solar irradiance of the Earth's surface peaks within the wavelength range we call "light".[9] That wavelength range also being the range we can see is just due to it being in our evolutionary environment: We have evolved to be able to see that range, because that's the range for which there was the greatest evolutionary pressure to be able to see. Red Act (talk) 08:06, 26 November 2009 (UTC)[reply]

Let me try to address the question in a different way. Einstein did not start out by assuming that the speed of light is constant -- this was a consequence of his theory, not an assumption. What he really cared about was Maxwell's equations for electromagnetism. According to Newtonian physics, Maxwell's equations could only be valid in one frame of motion (called the "ether"). This violated Einstein's intuitions: he thought that Maxwell's equations should remain valid even in moving reference frames, and worked out a set of rules that would make that happen. The constancy of the speed of light was a consequence of those rules. Looie496 (talk) 18:02, 25 November 2009 (UTC)[reply]

KageTora, the Universe has a speed limit and light happens to travel at that speed limit (because light particles are massless) so it is convient to call that speed limit the speed of light. You could also call it the speed of gravity which also travels at the speed limit. Dauto (talk) 19:28, 25 November 2009 (UTC)[reply]

November 25

Monstrous shape-shifting owl

I've just been sent a link to this YouTube video, which I found really freaky and disturbing, for some reason that I can't quite place my finger on.

http://www.youtube.com/watch?v=Es52WQKLumI

Does anyone know if this owl is actually a real species and can really do this? I'm thinking that while the first display might be real, the second (demonic looking) one is CGI.--84.69.185.215 (talk) 00:38, 25 November 2009 (UTC)[reply]

That was really cool. I don't know if it is real or not. I like it. Dauto (talk) 01:02, 25 November 2009 (UTC)[reply]

This has come up on the Reference Desk before, see Wikipedia:Reference desk/Archives/Science/2009 July 26#Cat bird owl?. Looie496 (talk) 01:11, 25 November 2009 (UTC)[reply]
Northern White-faced Owl also references that video. SteveBaker (talk) 03:57, 25 November 2009 (UTC)[reply]
Remarkable as it is it is quite small beer when compared to some of the courting postures of some Birds of Paradise of Asia.This is just one example. [10] Not much of a song but what a super blue smiley face. Richard Avery (talk) 08:46, 25 November 2009 (UTC)[reply]
Some more about the species here:[11] Here's pics of it in action:[12][13] Other "scops-owls" adopt a similar thin upright posture with ear tufts.[14][15][16] Fences&Windows 18:33, 25 November 2009 (UTC)[reply]
The owl reminds me (facially) somewhat of a Ring-tailed Lemur during the first transformation. It's probably not intentional - but I just thought that I'd make that observation. I felt slightly unsettled myself when I saw the owl's 'demon' form (anyone have any thoughts as to what it's trying to imitate with that?) - which may indeed be *exactly* what the owl wants you to feel. It's something about the way that its eyes appear to change shape that doesn't rest easy with me. --Kurt Shaped Box (talk) 01:34, 26 November 2009 (UTC)[reply]
The resemblance to a cat is striking, and a photographer described them as giving a "rarely heard cat-like cry",[17] Some kind of Batesian mimicry might be going on as an adaptation of the defence posture, but that's speculation. The general significance of ear tufts in owls has been discussed on a science blog here, and mimicry of mammalian predators is one suggestion. But other owls do adopt a similar posture, e.g. the Northern Pygmy-owl, in which it is described as a 'concealing' posture,[18] so the appearance to humans of the Northern White-faced Owl could just be a coincidence. Fences&Windows 12:57, 26 November 2009 (UTC)[reply]

Early Hominina finding in Americas

What is the oldest known Hominina remains found in Americas before the Homo sapiens arrived here? 174.114.236.41 (talk) 03:12, 25 November 2009 (UTC)[reply]

I find this: Models of migration to the New World. Bus stop (talk) 03:16, 25 November 2009 (UTC)[reply]
That article is about Homo sapiens, the OP is explicitly asking about pre-Homo sapiens. --Tango (talk) 03:23, 25 November 2009 (UTC)[reply]
There were no hominids in the Americas before Homo sapiens. The only primates in the Americas before the arival of Homo sapiens were the New World monkeys, and they are among the most distantly related primates to Homo sapiens around (far more distantly related than any other apes or monkeys). The first Hominids to arrive in the Americas were fully modern Homo sapiens. See Models of migration to the New World. There is some controversy as to the actual date the first people arrived; or from which direction, or whether they arrived in a single migration or in multiple waves, but even the oldest possible date for humans arriving is about 50,000 years ago, and that date is considered pretty old by most mainstream theories, which hold a date of about 17,000 years ago as the first arrival of humans. For comparison, the last non-Homo sapiens hominid, homo erectus, died out some 1,000,000 years ago. There is simply zero fossil evidence for any pre-homo sapiens hominids in the Americas; indeed as noted above, the nearest relatives are actually quite distant, those New World monkeys branched off of the "human family tree" well before any Apes or Old World monkeys did, some hundreds of millions of years ago. --Jayron32 03:26, 25 November 2009 (UTC)[reply]
There is some dispute about which was the last non-Homo sapiens hominid - it depends on whether you consider Neanderthals to be a subspecies of Homo sapiens or not (the experts can't agree). If you consider them a separate species then that moves to date from which Homo sapiens have been alone to, possibly, 30,000 years ago (although probably more). However, they were in Europe, not the Americas. --Tango (talk) 03:30, 25 November 2009 (UTC)[reply]
There is also Homo floresiensis, aka. "The hobbit", which may or may not have been a subspecies of Homo sapiens or a distinct species. Given that there are no actual extant hominids besides homo sapiens, it is hard to say whether any of these examples are "subspecies" or "species" - level relatives, it probably has a lot more to do with politics than real science as to which side of the debate one falls. Still, no fossil evidence of any of these hominids, or indeed of any other, before the arrival of modern Homo sapiens. --Jayron32 03:33, 25 November 2009 (UTC)[reply]
(Just today - and reported right there on the Wikipedia front page - is news that some rather convincing new studies show that florensiensis was indeed a separate species.) SteveBaker (talk) 04:31, 25 November 2009 (UTC)[reply]
Well, there ya go. But floresiensis wasn't in the Americas either. --Jayron32 04:36, 25 November 2009 (UTC)[reply]
Indeed not. SteveBaker (talk) 04:57, 25 November 2009 (UTC)[reply]

Hood Canal "tide computer"

Tide predictor, 1921

I came across this interesting snippet in the 'forum' posts at the end of this:

"...until very recent times the Ptolemaic theory was sufficient to calculate tide tables and it was a whole lot easier to use it rather than to puzzle out multi-body solutions with Newton's formula. The ancient Ptolemaic tables were good enough. At the Hood Canal locks here in Seattle there used to be a giant analog computer consisting of a heavy chain about fifteen feet long. The chain was fixed on one end and stretched horizontally with a series of pulleys representing the celestial bodies each able to indent the stretched chain and make it shorter or longer. The free end culminated in a pointer that moved up and down a scale indicating the expected tide."

I can't find anything about this on the web - does anyone have any more information about it? Hood Canal has nothing about it. Tide-predicting machine has a lot of discussion of these kinds of gizmo but nothing there sounds like it would have fifteen feet of "heavy chain" - it seems to discuss relatively delicate-looking gadgets.

SteveBaker (talk) 03:48, 25 November 2009 (UTC)[reply]

Um...since the Hood Canal isn't really a canal, but rather a fjord, why would it have locks? --jpgordon::==(o) 05:32, 25 November 2009 (UTC)[reply]
Wow. I did find the thing. "Old Brass Brains", they called it; it was used until 1965! Don't know the Hood Canal connection. This is from Technical World magazine in 1921; it's on page 205. It's a big download! There's also a modern writeup about it and others here. So cool! (--jpgordon::==(o) 06:21, 25 November 2009 (UTC)[reply]
Of course, I could have just looked more closely at tide-predicting machine. But it was fun research. Anyway, "Old Brass Brains" was big! It weighed 2500 pounds and was 11.5 feet tall. So "heavy chains" might be accurate. --jpgordon::==(o) 06:33, 25 November 2009 (UTC)[reply]
I had looked at the "Old Brass Brains" thing before I posted the question - but I didn't see the Hood Canal connection - and big though it is - it didn't seem like it would have 15 feet of "heavy chain" - and what chain it has isn't "stretched horizontally", it seems to zig-zag through the mechanism. It's clear that the person who I quoted (above) didn't understand the machine. Unless the one he's talking about is very different from the "normal" mechanical tide predictors, it would be summing an empirically derived Fourier series to make the predictions - not employing some kind of ancient Ptolomaic theory of how sun, moon and planets move as implied by "pulleys representing the celestial bodies". Mathematically and mechanically, the results might be kinda similar - you're adding circular motions - but "Old Brass Brains" certainly isn't using Ptolomaic principles. That's why I wanted to find out more about this supposed Hood Canal machine in Seattle. SteveBaker (talk) 14:30, 25 November 2009 (UTC)[reply]
Yeah. But as I said -- there don't appear to be any locks in the Hood Not-really-a-canal. --jpgordon::==(o) 15:38, 25 November 2009 (UTC)[reply]
Why would anyone make such a large tidal calculator as Old Brass Brains and not scale it down to a portable device like the Antikythera mechanism? How many of the Fourier series coefficients (up to 24 on one machine) correspond to identifiable astronomic influences? Cuddlyable3 (talk) 18:37, 25 November 2009 (UTC)[reply]
Seattle has Chittenden Locks - I don't think I'd describe them as being on the Hood Canal - but they are close enough that maybe someone could assume that they were. Still no sign of a tide predictor there though. SteveBaker (talk) 23:38, 25 November 2009 (UTC)[reply]

Type of truss bridge

Is the upper drawing a Warren truss bridge? I would guess so from attempting to read the description (I don't know French), but it differs from the lower drawing, which we currently use to illustrate the Warren section of truss bridge — the upper drawing has vertical supports, which are entirely missing from the lower drawing. If it's not a Warren, what is it? Nyttend (talk) 04:54, 25 November 2009 (UTC)[reply]

By the way, the reason that I'm asking is that I'm trying to ascertain the type of bridge shown in the photo. Nyttend (talk) 05:14, 25 November 2009 (UTC)[reply]


(ec) List of truss types identifies the lower image as a Warren (non-polar) truss - which agrees with the main article. The upper image does not appear in any of those types because the diagonal elements alternate in direction - unlike a Howe truss - and has verticals, unlike a Warran (non-polar) truss. The (french) caption says "Warren avec montants" - "avec" means "with" - so this is a Warren truss with "montants". My wife (who is French) says that "montants" are the vertical elements in window frames (the horizontal bits are "traverses")...and Wiktionary says that a montant is any vertical piece of carpentry in a framework. So evidently, your bottom image is a bona-fide Warren truss - and your top image is a modified version of a Warren - with those vertical bits (the montants) added in for whatever reason. SteveBaker (talk) 05:17, 25 November 2009 (UTC)[reply]
Well, now you've confused things - the photo you just added has only every second "montant" present! So that's yet another variation! I'm guessing though that this is still a Warren truss. SteveBaker (talk) 05:19, 25 November 2009 (UTC)[reply]
Briefly skimming an old engineering book I happen to have here at home - the critical thing about a Warren truss is how the weight is distributed between compressive and tensile forces in the diagonal elements. The Howe truss has all of the diagonals in the left-hand half of the span going from bottom-left to top-right and all the ones on the other side going the other way. A Pratt truss does the complete opposite - and which one you use depends on the building materials you have (are they better in tension or compression?) and whether you expect the truss to be fairly evenly loaded along it's entire length - or have a small, heavy load at just one point along the truss. It seems that the Warren design (by alternating the diagonals) is some kind of compromise between the two. The vertical elements...I have no clue...they seem to play little part in the discussion of the relative merits of the three designs - but so many real bridges seem to have them that they must be important in some manner. SteveBaker (talk) 05:32, 25 November 2009 (UTC)[reply]
Thanks for the details; I've never quite understood the different types of trusses, so I wasn't at all sure that the montants weren't significant for the type of truss. If you're an engineer, you may find this location quite interesting: it's the Deep Cut of the Miami and Erie Canal. Nyttend (talk) 05:47, 25 November 2009 (UTC)[reply]
By the way - I doubt that "montant" is the right word for the upright beams in bridge structures. It's evidently the right word in French - but probably not in English. I was merely using the word as a shorthand for whatever the correct term is! SteveBaker (talk) 06:01, 25 November 2009 (UTC)[reply]
According to this [19], it is a "varying depth Warren truss". --Cookatoo.ergo.ZooM (talk) 09:11, 25 November 2009 (UTC)[reply]
The varying depth refers to the polygonal arrangement of the chords, not the upright posts. We are still looking for a name for a Warren truss with posts. SpinningSpark 12:59, 25 November 2009 (UTC)[reply]
This page and this page call it a "subdivided Warren truss". If you want a fancy architectural term for the vertical members, you could call them mullions. Gandalf61 (talk) 13:25, 25 November 2009 (UTC)[reply]
Yeah, doesn't seem to have an actual name, just Warren truss with verticals seems the most usual description. The vertical members, according to this book, are called posts and hangers depending on whether they are taking a compressive or tensile load. SpinningSpark 13:31, 25 November 2009 (UTC)[reply]
Mullion is a term used in connection with windows and doors and other openings. The side of a bridge is not intended as an opening (either for people of light) so I think it would be unlikely to be used. SpinningSpark 13:41, 25 November 2009 (UTC)[reply]

Dehumidifiers and climate change control

Given water vapour's role in the greenhouse effect, could large-scale dehumidifiers be used to offset emissions of other greenhouse gases and control climate change? NeonMerlin 05:25, 25 November 2009 (UTC)[reply]

No. --Jayron32 05:27, 25 November 2009 (UTC)[reply]
Well, yes and no. Water vapor is itself a greenhouse gas - so I suppose a dehumidification of the upper atmosphere would help to alleviate the greenhouse effect. However, water droplets in the form of clouds and fog banks are very bright and reflect sunlight back out into space - keeping the earth cooler. So in that regard, getting rid of the clouds would make matters worse. (This was noted in the days after the 9/11 terrorist attacks when all North American airliners were grounded. The lack of contrails - which are mostly white water vapor - caused a noticable increase in temperatures throughout the USA.)
But either way, it's crazy to imagine that we could build dehumidifiers on a large enough scale to make any impact without burning so much energy to drive them that our net carbon footprint would go through the roof!
So "No" for a couple of reasons. SteveBaker (talk) 05:40, 25 November 2009 (UTC)[reply]
If you could then you could water the entire Sahara with the water taken out of the atmosphere...--BozMo talk 07:10, 25 November 2009 (UTC)[reply]
Also, don't dehumidifiers generate a lot of heat in their operation? I suspect the heat generated would be more than the heat allowed to escape into space by the reduced water vapour in the air, even without factoring in the albedo effects. --Tango (talk) 23:32, 25 November 2009 (UTC)[reply]
Unlike other greenhouse gasses, water vapor is fairly self-regulating: on average, the atmosphere is fairly close to saturation, so any increase in evaporation will be countered by an increase in rainfall. --Carnildo (talk) 02:02, 26 November 2009 (UTC)[reply]

do blue tulips occur in nature?

Tulipa sylvestris

do blue tulips occur in nature?

Define "natural". Tulips are a highly cultivated genus of plants, see tulip. Like maize and other highly cultivated plants, it is doubtful if you have ever seen a truly "wild" tulip before (you may have seen feral tulips, but that is different than a truly wild species). There is apparently a variety called "Tulipa sylvestris" which is known as the "wild" tulip; however we have no article on this, so I have no reference for what "natural" tulips look like. --Jayron32 05:34, 25 November 2009 (UTC)[reply]
Type "Tulipa sylvestris" into Google Images and you'll be greeted by a ton of bright yellow flowers with petals that are much more open than the classical idea of what a Tulip should be. So, no - no blue tulips in the wild...or red ones, or orange ones...just yellow. SteveBaker (talk) 05:55, 25 November 2009 (UTC)[reply]
Photoshop trumps nature. Cuddlyable3 (talk) 08:20, 25 November 2009 (UTC)[reply]
Eeew!! Are you certain about that? ;-)) Richard Avery (talk) 08:33, 25 November 2009 (UTC)[reply]
There are a few cultivars that approach blue: for example 'Blue Parrot', seen here [20]. As for naturally occuring species (about 150 [21] ), I don't think there are any blue species, although there are certainly purplish species. --Eriastrum (talk) 00:08, 26 November 2009 (UTC)[reply]

North Sea gas

InThe Death of Regginald PerrinbyDavid Nobbs, Jimmy attempts suicide by putting his head in the oven, and Reggie laughs at his ineptitude: "North Sea gas isn't poisonous." I thought this was either a bizarre blunder by the author, or exceedingly obscure humor. But now I'm reading a story by P. D. James in which a murder attempt fails: "The conversion, that's why. We're on natural gas from this evening. That North Sea stuff. It isn't poisonous. The two men from the Gas Board came just after nine o'clock."

What the heck do they pump out of the North Sea? Hydrogen? —Tamfang (talk) 05:28, 25 November 2009 (UTC)[reply]

North Sea Gas (aka "Natural" gas) is mostly Methane - which isn't poisonous. If the air gets too full of the stuff - it would eventually displace all of the oxygen and you'd suffocate - but that's not really likely from a gas oven. The biggest concern would be explosions. The old "Coal Gas" or "Town Gas" that was used in the UK before the North Sea oil boom was a mixture of hydrogen, methane and carbon monoxide. The carbon monoxide would kill you pretty quickly. I'm pretty sure the UK was in that transition period right when Reggie Perrin was imagining his mother in law as a hippo - so it's perhaps understandable that someone would still attempt suicide that way if they didn't know any better. Maybe the advertising of the benefits of the new gas was highly prevalent at the time the series was made - so perhaps the implication was that the guy who was attempting this was an idiot for not knowing about that. SteveBaker (talk) 05:50, 25 November 2009 (UTC)[reply]
For the record, natural gas is not a term that applies just to "North Sea Gas" — reading through the article, "natural gas" appears to be a super-generalization, describing many variants. Comet Tuttle (talk) 19:02, 25 November 2009 (UTC)[reply]
I was an accident investigator for Shell when Tokyo switched. Funnily the pattern of failed suicides (booze, oven, wake up with headache) was sometimes followed by lighting a cigarette and starting a non fatal gas explosion. When I left they were looking at smart meters to contact the police for suspected suicide. It is similar to using car exhaust to kill yourself when you have a Cat Converter on the car. --BozMo talk 07:03, 25 November 2009 (UTC)[reply]
Oh, there goes that plan. Thanks, all, for the clarification. —Tamfang (talk) 07:07, 25 November 2009 (UTC)[reply]
At the time of the changeover there was a lot of public debate over North Sea gas being odourless. I believe something is added to it to give it a human detectable odour but it is very mild compared to the strongly sulfurous smell of the old coal gas and this led to some public concern. It was a stock response from industry and government spokesmen that there was no need to do anything about this because the gas is not poisonous. The issue was therefore likely to be in people's minds at the time and they would have got the joke. SpinningSpark 12:23, 25 November 2009 (UTC)[reply]
Seriously? "Not poisonous" is not much comfort when your house goes up like a bomb. Was there an uptick in explosions corresponding to the downtick in poisonings? --Trovatore (talk) 18:15, 25 November 2009 (UTC)[reply]
I doubt that - if someone dies from carbon monoxide poisoning with their head in a gas oven - they are unlikely to turn it off afterwards! Hence, probably, the number of explosions would be pretty similar. But once people knew that this was not a viable suicide technique - the rate of explosions due to that ought to have dropped substantially. SteveBaker (talk) 23:01, 25 November 2009 (UTC)[reply]
Do they not use Butanethiol in the UK? --Jmeden2000 (talk) 16:04, 25 November 2009 (UTC)[reply]
Yes, I think that's what is used, and I know it is a very nasty chemical in high concentrations. But it is used in tiny quantities and definitely does not smell anywhere near as repulsive as coal gas. One whiff of that had us immediately opening all the windows to dispel it. Natural gas simply does not cause that same reaction in people. At least, with the additives used in Britain, I could not speak for elsewhere. SpinningSpark 17:35, 25 November 2009 (UTC)[reply]
[This] link claims the UK smell is due to a blend of mercaptans and sulphides. But a trace of any thiol is going to smell enough to detect. I would suspect a blend is used so that the gas detector vans can check the ratio of the odourants and confirm a gas leak - it might be that a nearby plant could be using one of the components, but the ratio in the detector would then be different. LPG uses ethanethiol to make it smell - I think the butanethiol is too high boiling here - it would tend to concentrate towards the end of the bottle - I know that the ethanthiol has a small tendency to do that - and I can tell (on the boat) when the bottle is starting to get low by the smell of the cooker.  Ronhjones  (Talk) 19:35, 25 November 2009 (UTC)[reply]
Someone should really move that article to butyl mercaptan in accordance with WP:COMMONNAME. In my opinion the chemistry editors are overly respectful of IUPAC's silly nomenclature. But I'm not a chemist so I won't do it myself. --Trovatore (talk) 18:12, 25 November 2009 (UTC)[reply]
See a full account in Kreitman, Norman (1976). "The coal gas story. United Kingdom suicide rates, 1960-71". Br J Prev Soc Med. 30 (2): 86–93.. Fences&Windows 18:02, 25 November 2009 (UTC)[reply]

overies

wat happens if the size of overies s larger??? —Preceding unsigned comment added by Shilpa.upadhya (talkcontribs) 07:37, 25 November 2009 (UTC)[reply]

A common cause of swollen ovaries is a cyst or PCOS. We have an article about PCOS[22] and there is more information on the web.Cuddlyable3 (talk) 08:14, 25 November 2009 (UTC)[reply]

Balancing chemical equations

Is there any real, concrete method for balancing chemical equations? If not, can anyone offer any advice or tips? Thank you. —Preceding unsigned comment added by 161.165.196.84 (talk) 08:51, 25 November 2009 (UTC)[reply]

There must be such methods, because automatic equation balancers exist, e.g. http://www.webqc.org/balance.php.
Does Chemical equation#Balancing chemical equations help?
A good tip is to start by balancing one particular element (i.e. making sure there are equal numbers of the chosen element on both sides of the equation) and then putting in enough of the other elements to make the required product.
Ben (talk) 11:54, 25 November 2009 (UTC)[reply]


Chemical equations can be written as simultaneous equations requiring integer solutions. Written in that form, any linear equation solver like Gaussian eliminationorlinear programming, or a complicated optimization method, can be applied. Finally, for convenience, the result is usually reduced by dividing out any common integer factors, if the solution method doesn't already take care of this. Nimur (talk) 13:34, 25 November 2009 (UTC)[reply]
The best tip I give my students is that, if done correctly, the last element always balances itself; so if there is some element which is complicating the solution because it appears in multiple compounds on each side of the equation (oxygen does this often), leave THAT element till last, and it will work itself out. Additionally, some equations may seem to never balance; my suggestion in these cases is to consider using fractional coefficients and then multiply out to get rid of the fractions. Just a few tips. --Jayron32 04:28, 26 November 2009 (UTC)[reply]

How does an increased heart-rate stimulate the body's metabolism?

Obviously, excersise raises one's metabolism (although scientifically speaking, I'm not sure why), but how does just having an increased heart-rate (perhaps by consuming caffiene)stimulate the metabolism? Thank you.161.165.196.84 (talk) 09:27, 25 November 2009 (UTC)[reply]

(For the impact of exercise on the body, see Exercise physiology.) I don't know the actual biochemistry involved, but by definition, the energy used by the heart would increase if its output were to increase. Zain Ebrahim (talk) 14:09, 25 November 2009 (UTC)[reply]
I think we need to start by asking whether the body's metabolism is stimulated by increased heart rate. There are many ways to increase the heart rate, and many of those could affect metabolism in ways that don't directly involve the heart. One way to separate the effects would be to use an implanted cardiac pacemaker to increase the heart rate directly. Speeding up the heart above normal rate using a pacemaker is called ventricular overdrive pacing redlink!, and is used diagnostically and therapeutically in tachycardia. I haven't found a reliable source for this (yet), but my own experience suggests that metabolism is not globally increased during this maneuver. It is clear that cardiac metabolism is increased (PMID 8261221), as Zain has pointed out. -- Scray (talk) 14:24, 25 November 2009 (UTC)[reply]
exercising raises one's metabolism for that particular part of the body exercised, there is increase of heart rate and respiratory rate (by both neurological and hormonal activation), with corresponding increase in metabolic activity in terms of cardiac muscle as well as the respiratory muscles. while doing exercise, organic acids and products of anaerobic respiration such as lactic acid will be released into circulation, and these will be metabolised by the liver, and this would lead to increase in metabolic rate in liver as well. bubu~ (talk) 15:29, 25 November 2009 (UTC)[reply]

Freezing contents of deeply chilled liquid.

What happens to a deeply chilled, but unfrozen drink; that in the few seconds,usually a bottle container, is opened, the liquid freezes immediately? In the many discussions held about this phenomenon, none could come up with a convincing answer. Some tried to say that when the oxygen enters the bottle container, it makes the liquid to freeze. It was not convincing since the oxygen entering the top of the bottle is warmer than the inside of the bottle. Please help41.17.171.15 (talk) 12:00, 25 November 2009 (UTC)[reply]

The liquid has been supercooled but freezing will not start unless there are small particles present that allow nucleation to begin. Opening the bottle releases bubbles which provoke the nucleation process. SpinningSpark 12:07, 25 November 2009 (UTC)[reply]
It could be that, or it could be that the phase diagram of water indicates that there is an inverse relationship between pressure and phase for water; that is water under higher pressure tends to be more likely to be a liquid, and it actually freezes as the pressure decreases. Additionally, the released gas can cause adiabatic cooling in the liquid (open a can of soda, its temperature will drop slighlty but measurably as the bubbles are released). I think it is some combination of these three effects; though I think that Spinningsparks answer on supercooling is likely the dominant effect here, the other two effects may play some role as well. --Jayron32 04:24, 26 November 2009 (UTC)[reply]

Finding power factor

Suppose I find the time delay between the voltage and current waveforms and divide that by the time period of half a cycle. Then subtracting it from 1, won't that give me the power factor? If not, please suggest how to. I want to avoid finding out phasor angular difference. 218.248.80.112 (talk) 14:18, 25 November 2009 (UTC)[reply]

The time delay to the crest of the current sine wrt the crest of the voltage sine is only part of the power factor calculation (namely the leading/lagging portion) you still need to know the total power drawn vs the peak of the current sine (apparent power) to figure out what the number should be. If the current sine is perfectly identical to the voltage sine this might be a simple process (mathematically) if not you will need to perform some advanced calculations. Hope this helps! --Jmeden2000 (talk) 15:56, 25 November 2009 (UTC)[reply]
What Jmeden2000 is alluding to is that the power factor may vary if your current and voltage waveforms aren't perfect sinusoids. If they are not, then all bets are off, and simple power-factor formulae are useless - you will need to perform a full integral over the waveforms, .
In any case, your approach will yield:
This is approximately, but not exactly equal, to (the correct value of the power factor), for sufficiently small and perfect sinusoids. I think you're missing a factor of pi, too. You probably meant:
...which is sort of like a first-order taylor series approximation of a cosine. (But not a correct one).
The degree of applicability of this engineering approximation will depend on your application's precision needs. I can't think of any engineering application today where this approximation would be useful, because calculating the true phase difference and computing its cosine is trivial on even the most reduced forms of computers, microcontrollers, or even analog signal conditioners. Nimur (talk) 17:41, 25 November 2009 (UTC)[reply]
After re-reading your question, it sounds like you just want to avoid using phasors. That's fine - but your approach has already calculated the angular difference in the time-domain (without using phasors): . Is there any reason you don't want to use cosine? Nimur (talk) 18:01, 25 November 2009 (UTC)[reply]
I had to skip all those integrations and all because I am implementing this on an 8051 microcontroller which does not have any floating point capability and only 8bit math. So that was also the reason of not producing the cosine.. I think I will extend this project to simple power factor improvement (like connecting a capacitor through relay for low power angle) and protection (cutting of the circuit for very low power angle). Any suggestions ? And btw thanks for the analysis.. 218.248.80.112 (talk) 09:50, 26 November 2009 (UTC)[reply]
Fixed point math throws some kinks in to things, but you can still compute cosine with a lookup table. But, if your only actions are to throw a relay or two, you may as well just solve for the phase difference that you want to use as trigger points (instead of power factor). You might also want to be aware that simply throwing a relay to change the circuit may result in a phase oscillation - for example, you may be right on the edge of an acceptable phase error, throw the relay, and then fall just on the other side, un-trigger the relay... in other words, unless you analyze the closed loop circuit, including your software logic, you may have created an unstable circuit. The correct way to design such a power factor correction circuit is a little bit more complicated. For example, take a look at Fairchild Semiconductor's Application Note AN-42047 - Power Factor Correction Basics. They detail the theory and practice of PFC circuits, and pitch some of their products that do this for you. Nimur (talk) 15:12, 26 November 2009 (UTC)[reply]
Certainly you could use a lookup table for cosine - and if you need higher precision and/or are too limited on memory to get the precision you desire then you can dramatically improve the results by doing a simple linear interpolation of two consecutive table entries. SteveBaker (talk) 17:33, 26 November 2009 (UTC)[reply]
If you do what you said, but divide by the time of a complete cycle, then multiply by 360, you will be finding the phase difference in degrees between the waveforms. Power factor is defined as the cosine of the phase diff.
If you really dont want to know or find the actual angle then since P = VIcos(phi), you could measure the average power, rms voltage and rms current, then substitute to find cos(phi), which is the aforementioned power factor —Preceding unsigned comment added by 79.67.32.122 (talk) 00:49, 27 November 2009 (UTC)[reply]

Ionic food preserver

Does anyone have any idea how this "ionic preserver" works to prevent mould growth, banana pigmentation, and the various other food spoilages that they are claiming? I searched Google but couldn't find a thing. The only clue from the food preservation article is nitrites and sulphites, but surely the device would run out of them eventually and there is no mention of a plug-in pack or the like. --Mark PEA (talk) 17:02, 25 November 2009 (UTC)[reply]

Ionising air will produce ozone. The Wikipedia article on ozone says it can be used as a disinfectant for killing bacteria on food and to kill mold spores. I'm not sure if this is the specific mechanism for the "ionic preserver", or if the preserver would produce a sufficient concentration of ozone to do anything.
Here is a scientific paper saying that negative ions preserve lettuce; they seem to reduce water loss (which would cause drying). --Maltelauridsbrigge (talk) 17:26, 25 November 2009 (UTC)[reply]
Ozone reduces the ripening of bananas by reacting with, and hence eliminating, ethylene which is the chemical that causes bananas to ripen.[23][24][25]
Air ioniser and Air_purifier#Air_ionizers_and_ozone will tell you more about this idea in general, but not this specific product. The idea has been around since at least the 1950s.[26] Negative ions and ozone may restrict the growth of microorganisms.[27] Here's a study of the use of an ionizer in a domestic fridge that says it is effective:[28] Fences&Windows 17:51, 25 November 2009 (UTC)[reply]
Okay, so basically it is 3O2 -> 2O3. Aerobes don't proliferate, cheese doesn't dry out, etc. I do wonder about the safety of the product though, I'm guessing there aren't any longitudinal studies looking at respiratory diseases of those who have these in their fridge. --Mark PEA (talk) 18:35, 25 November 2009 (UTC)[reply]
It's a UK advert - which is good, because it will have to tell the truth (or the ASA will jump on them like a ton of bricks) - it's main claim as I see it is "it reduces the damaging gases involved in food decay" - I would guess that it will keep down the concentration of ethylene, that would slow down any ripening process, and if the food doesn't over-ripen, it won't go bad so fast.  Ronhjones  (Talk) 00:45, 26 November 2009 (UTC)[reply]

Ozone is a strong oxidiser and will cleave unsaturated compounds by ozonolysis. Ozone will react with ethylene (possibly into formaldehyde units) cuz of the unsaturated pi bond. The side effect of applying ozone for too long I think is rancidification. John Riemann Soong (talk) 13:03, 26 November 2009 (UTC)[reply]

November 26

age of atoms

Is there a difference between an atom of carbon when it was created billions of years ago and one now that is billions of years old? In other words are carbon atoms mortal and if so over what time scale do they show any change? 71.100.11.112 (talk) 04:44, 26 November 2009 (UTC) [reply]

Well there are different types of carbon atoms, or of any atom, called isotopes. According to our article Isotopes of Carbon, two of these isotopes are stable, meaning that they will last forever under ordinary conditions. The rest of the isotopes are "mortal" or unstable, the most famous of which is probably Carbon-14 which has a half-life of about 5703 years. When it "dies" or decays, it becomes a Nitrogen-14 atom, which is stable. Jkasd 04:59, 26 November 2009 (UTC)[reply]

Interesting...and what decays to become stable Carbon-12, or are we stuck with however many of them we started with? DRosenbach (Talk | Contribs) 05:02, 26 November 2009 (UTC)[reply]

I think that most of the Carbon-12 in the universe is due to the CNO cycle in stars. Jkasd 05:07, 26 November 2009 (UTC)[reply]
I am certainly no expert, but the answer to your question is "it depends." If the carbon atom is an unstable isotope (radioactive), it will spontaneously decay at a time, which in my understanding is quite random. There is a way to measure this decay, but it is not dependent on an individual atom. The measurement assumes that you have a mass of multiple unstable atoms of the same type. When half of the atoms have decayed (changed form into another atom), that is called the half life. Carbon decay is specifically significant; look at carbon dating, where people attempt to calculate age by what percentage of the estimated original unstable carbon isotopes remain in an object. As for the time scale, it varies from almost immeasurably small amounts of time to hundreds of thousands of years, depending on the atom. As for mortality, only living beings can be mortal, at least in my definition of the word. An atom, so far as anybody knows, has absolutely zero self awareness or thought process, and as such is abiotic. An atom is therefore neither mortal (it will die someday) or immortal (it will live forever). Also, theoretically if one returned the appropriate particles to a decayed atom, it would return to its unstable form (we can do that with some nuclear reactions). Finally, if you have two carbon atoms of the same isotope, but one is three billion years older than the other, I don't believe that there is any difference. I hope this gives you a start, and since I am not an expert I'm counting on other editors to correct me if I am wrong about something ;-). Falconusp t c 05:11, 26 November 2009 (UTC)[reply]
I think the talk of isotopes is a bit of a distraction. In either case, a billion year old atom is the same as a newly created atom. That is, a billion year old atom of C-12 is the same as a newly created one and a billion year old atom of C-14 (if you could find one) will be exactly the same as a newly created atom of C-14. So, no, they don't "age". The unstable ones do randomly decay, but not because they are old, just because of a random event happening. (Don't think of it like trees dying when they get old, but dying when they are randomly struck by lightning, regardless of age.) StuRat (talk) 05:30, 26 November 2009 (UTC)[reply]
Ah... So then are we speaking of conservation of something... perhaps conservation of stability? 71.100.11.112 (talk) 06:03, 26 November 2009 (UTC) [reply]
No, I don't think it is really a conservation law. It's about the indistinguishability of atoms of the same isotope. --Tango (talk) 09:28, 26 November 2009 (UTC)[reply]
See also radiocarbon dating.--Shantavira|feed me 06:57, 26 November 2009 (UTC)[reply]
Radiocarbon dating determines the age of a sample or population of atoms, not the "age" of an individual atom. Individual atoms, as StuRat and Tango said, do not age (in the case of unstable isotopes this means that an atom/nucleus just before it decays is indistinguishable from one that has just been created). --Wrongfilter (talk) 09:49, 26 November 2009 (UTC)[reply]

However, StuRat is correct in stating that I am not referring to radioactive decay of unstable isotopes but to aging of stable isotopes or aging of any stable particle. If you insist on including unstable isotopes then the question applies only to an isotope atom between the time it is created and the time it transmutes such as an atom of carbon-14 before transmuting to nitrogen-14. It is during the time the atom has not transmuted which concerns me and that I reference rather than to the transmutation itself. However, I leave the question open so you can answer it any way you will - transmutation included or not. 71.100.11.112 (talk) 10:19, 26 November 2009 (UTC) [reply]

In that case, no. There is absolutely no ageing of atoms. An atom of a particular isotope of a particular element is identical to any other, there are no changes over time. There are some changes in energy levels, but they are fluctuations, not any consistent change - they take place over nanoseconds (or smaller timeframes) and are insignificant over longer timescales. --Tango (talk) 10:34, 26 November 2009 (UTC)[reply]
What else in the Universe including the Universe itself does not grow older or undergo changes we can attribute to aging? 71.100.11.112 (talk) 10:49, 26 November 2009 (UTC) [reply]
Well, the universe itself certainly ages. It expands (ie. becomes less dense) and the Cosmic Microwave Background cools down (that is part of the expansion) and entropy increases. Atoms and sub-atomic particles don't age. Molecules don't really age, although some large ones change shape over time (although I don't know of any such changes that happen in a particular direction so could be considered ageing). Stars certainly age, as do geologically active planets and planets large enough not to have finished cooling and whole galaxies and clusters of galaxies. Small planets and asteroids pretty much stop ageing after a certain amount of time (a billion years, say). They may pick up a few extra impact craters, but that's about it, although I suppose the isotope ratios change over time as the time since the matter that built up the asteroid was created in a supernova increases. So, in conclusion, if you are willing to look very closely pretty much anything larger than an individual molecule probably ages in some way. To an extent, this is a matter of definitions, though - everything changes, but for some things we consider any change to make it a new thing. If we considered an atom that undergoes radioactive decay to remain the same atom, just in a different form, then even atoms age - as they get older, they move towards more stable forms. This is a very statistical form of ageing, there is no way of measuring the age of a single atom, but I suppose it would be a form of ageing. We usually consider it a new atom, though, so that is irrelevant. I apologise for the stream of consciousness - this was going to be a coherent answer when I started it! I hope it is still useful. --Tango (talk) 11:13, 26 November 2009 (UTC)[reply]
No need to apologize. The question begs the answer and I can imagine a stable particle aging completely within the entire duration of its creation especially if that time span is instantaneous. 71.100.11.112 (talk) 12:05, 26 November 2009 (UTC)[reply]
There are particles that are 'immune' to time: massless particles like photons that travel at the speed of light and therefore experience no time. Fences&Windows 12:16, 26 November 2009 (UTC)[reply]
Massless particles seem to be problematic: c = (1/M)*(M*E)^(1/2) (excuse the plain text form) 71.100.11.112 (talk) 13:06, 26 November 2009 (UTC) [reply]
I'm not sure where that formula came from, E=mc^2 rearranges to c=(E/m)^(1/2). While that looks problematic, that formula is for rest energy and massless particles can never be at rest, so the problem never comes up. If you interpret m as relativistic mass then that formula does give you the energy of the photon (but only because the relativistic mass is calculated using that formula!). --Tango (talk) 13:33, 26 November 2009 (UTC)[reply]
The article you need to read, I think, is indistinguishability, at least up to the point before it launches into all the wave mechanics notation. When discussing particles at the quantum level, the concept of individual particles with individual identities starts to lose its meaning. We cannot tell, even in principle, which particles are the oldest because separated particles are a misconception. It is a bit like asking which is the oldest wave in the sea. SpinningSpark 12:54, 26 November 2009 (UTC)[reply]

Blackholes don't age. Dauto (talk) 13:12, 26 November 2009 (UTC)[reply]

They decay - see Hawking radiation. They also absorb background radiation (which any star sized or larger black holes will do more of at the moment than they radiate). --Tango (talk) 13:33, 26 November 2009 (UTC)[reply]
Yes, they decay, but as pointed out above by others, decaying is not the same thing as ageing. In other words: an old blackhole is indistuinguishable from an young one, as long as they have the same mass, charge, and angular momentum. see the no hair theorem. Dauto (talk) 14:26, 26 November 2009 (UTC)[reply]
True. Given one black hole there is no way to determine its age since you don't know the starting size. However, given the same black hole at two different times if is possible to determine which is the older, which can be interpreted as a kind of aging. --Tango (talk) 15:32, 26 November 2009 (UTC)[reply]
Having recently sat through a NASA lecture about interplanetary petrology, I may have some insights. The definition of "age" of a "thing" is always a little ambiguous when dealing with cosmogenic timescales. Technically speaking, (actually, not technically speaking at all), according to our best understanding of the universe, everything always existed and always will exist (although even this one gets debated by the really nitpicky cosmologists. So when you want to know the "age of a thing", what you really are asking is "how long since ____ happened to these atoms?" The blank can be filled in with a lot of complicated particle physics, geology, etc. The actual atoms are the same ones that came out of stars (usually); but even if they are not, the subatomic particles they are made of are the same ones that came out of stars; and so on. So, Instead of asking the "age of an atom," it is really better to ask "how long has this rock contained these molecules", "how long has this molecule contained these atoms", "how long since this atom mixed with these other atoms that are in the same molecule," or "how long since the protons of this atom all stuck together?" These are more precise questions and they're much easier to get exact-ish years.
But, moving past some moot details, in the beginning there was a great infinitely dense mixture of undifferentiated energy and matter that obeyed the laws of General Relativity. All this muck swirled around in a sort of mind-bending cosmic singularity, but suddenly it started to break symmetry, and various types of subnuclear particles manifested themselves and obeyed different force laws, and it became possible to identify those which were matter-like and those which were particle-like carriers of forces that had just started to be identifiable. So, by now, all the matter and energy in the universe "existed" - and strictly speaking, everything is this old (13 billion years, often quoted with more precision in case you want to calibrate your calendar). At this early stage, nothing was an atom yet, let alone an identifiable hunk of space-debris or rock or self-locomotive, talking, carbonaceous life. Nonetheless, everything is "this old", insofar as its primordial cosmogenic ancestors came from this material.
Anyway, flashing forward a few hundred million boring years, past the formation of the proto-nuclei and arriving at the plasma-like single protons swirling around in space, we get to stellar formation of the first round, in which hydrogen squishes together and stellar nucleosynthesis produces the standard spectrum of nuclei (atoms) that are found in almost everything we observe in space. So from some standpoint, all the atoms are really this old (neglecting, of course, a tiny but relevant fraction of nuclei which were synthesized in Big Bang's first few moments before the expansion brought the density below critical levels). Many, but not all, of the atomic nuclei are this old.
A key thing happens here: probably, this star extincts itself, blows out all these atoms somewhere else, and they accrete in a new cloud or disc, the solar nebula or solar disc. These atoms are all floating around, but they begin to gravitationally self-attract, and finally compress in to start the formation of Sol. This is another unique age for all the rocks we find in space, because at this point, differentiation becomes relevant (e.g. the melting of rock by gravitational force). Gravity-based melting distills out the heavy and light nuclei, and so a benchmark of separated-nuclei concentrations is present. This is our "baseline" particle-ratio for heavy-nucleus radioisotope dating, because after this point, fractionation causes different rocks to have different concentrations of things. And once rocks sink into a convective melting process inside a planet, complete with metamorphic rock formation, all bets are off as far as where an individual atom will go.
Finally, once a planetary rock makes its way to the surface, the rock begins to weather and erode, and oxidize (depending on the planet). So we can see an exchange of material again taking place; and we can date the age of this interaction.
All told, it's really the same material getting mixed up over and over again; but at different timescales, different amounts of mixing happen. Over the lifespan of the universe, a single atom has progressed from an undifferentiated Planck soup into a Hydrogen atom into a complex nucleus into a fully-grown atom with its own electrons; and finally, it found other atoms and formed a molecule with those; and those molecules might have changed around a lot (and even the atomic nuclei might have shattered once or twice); and the large chunks of molecules probably convected a lot inside of a planet, and then probably eroded a whole lot at the surface; so the "age" is whatever you want to define it to be: the time since ____ last happened. Nimur (talk) 15:48, 26 November 2009 (UTC)[reply]
Are you saying that while the bricks don't age the mortar and the building does? 71.100.11.112 (talk) 20:26, 26 November 2009 (UTC) [reply]
I'm saying that the age of the building or the wall is not the same as the age of the brick. You could spin that into your summary, as long as you recognize the limits of the metaphor. Nimur (talk) 20:51, 26 November 2009 (UTC)[reply]

Both analog and digital possible?

Are there any FTA receivers which can receive both analog and digital satellite television? --84.61.167.221 (talk) 14:07, 26 November 2009 (UTC)[reply]

Sun looks different in size

Is there a scientific explanation for the reason we seen the Sun bigger in size during sunrise/sunset than in the mid-noon. I might be wrong but many have been asking this.--Email4mobile (talk) 14:53, 26 November 2009 (UTC)[reply]

See moon illusion. Gandalf61 (talk) 14:59, 26 November 2009 (UTC)[reply]
Thanks a lot, Gandalf61.--Email4mobile (talk) 15:09, 26 November 2009 (UTC)[reply]
It's a very powerful illusion. I work in computer graphics and one task I've been handling recently is drawing the sun and moon realistically in computer games. When I calculate the mathematically correct sizes to draw them, they look ridiculously small because that moon illusion works in reverse on a computer screen. I had to draw them both at four times the realistic size in order to please the majority of the 30 or so co-workers I polled for their judgement of the right size! I find this deeply disturbing to my sense of "doing it right" - so I eventually compromised at three times the correct size. SteveBaker (talk) 17:26, 26 November 2009 (UTC)[reply]
I've read the moon illusion, but couldn't get a final explanation for that phenomenon. Can you give me a conclusion for that?--Email4mobile (talk) 17:36, 26 November 2009 (UTC)[reply]
It's a surprisingly powerful illusion - nearly everyone overestimates the size of the moon - but you can use the "Can you cover the moon with the tip of your pinky finger at arms-length?" test (yes, you can) to convince yourself that sun & moon are indeed the same size no matter where they are in the sky. Basically - when the moon (or sun or whatever) is far above the horizon, you have no context for judging it's size - when it's close to the horizon, you have other objects to compare it against - since it's "behind" everything else, your brain registers that it's a long way away - since it's obviously larger than things like houses and trees - it looks gigantic (which is fair because it IS gigantic). SteveBaker (talk) 18:33, 26 November 2009 (UTC)[reply]
So it is a state of relativistic views rather than light intensity effect. Before, I asked and before reading about it, I was thinking it was due to different light intensity during the different times. Thanks SteveBaker.--Email4mobile (talk) 21:33, 26 November 2009 (UTC)[reply]
I wouldn't call it relativistic, I would call it relative - just to avoid ambiguity about the actual effect. Nimur (talk) 22:38, 26 November 2009 (UTC)[reply]
Right - the image at right really explains it well. It's called "The Ebbinghaus illusion". Imagine that the orange dot is the moon - but with the largeness of the sky it looks smaller than with the small details of the horizon. SteveBaker (talk) 23:32, 26 November 2009 (UTC)[reply]

How big can a Thanksgiving turkey be?

The Wikipedia articles on turkeys don't say how big a Thanksgiving turkey can be, and searching around on the Internet did not tell me, either. I know that the supermarket has turkeys larger than 24 pounds, but how big can they be? --DThomsen8 (talk) 14:56, 26 November 2009 (UTC)[reply]

Is there some reason Thanksgiving turkeys are different from normal turkeys? Googling suggests a world record was set at 86 pounds - Wikipedia blocks the link because apparently it's spam. Are you perhaps talking about specific weights at which turkeys are commercially sold? Vimescarrot (talk) 15:34, 26 November 2009 (UTC)[reply]
The human variety may weigh more. 71.100.11.112 (talk) 15:52, 26 November 2009 (UTC) [reply]
Googling around suggests that birds as large as 40lbs can be bought commercially - but almost all of the "How long does it take to cook" guides top out at 30lbs. Many of those guides recommend buying two smaller turkeys when you need more than 18lbs because it's easier to control the cooking process. SteveBaker (talk) 17:21, 26 November 2009 (UTC)[reply]
Turkeys have been bred over generations to be larger and larger. Today they are so big they can no longer mate on their own, and must be artificially inseminated. They are also starting to have problems walking. To make them larger would probably require too much human intervention to make it cost effective. Ariel. (talk) 19:30, 26 November 2009 (UTC)[reply]
With the above mentioned cooking control issue it seems like even large kitchens might prefer smaller turkeys. Perhaps GM will find a way to make legs stronger or to eliminate unwanted parts. I dare to imagine what kind of "turkey" science could come up with if it really went to work. 71.100.11.112 (talk) 19:39, 26 November 2009 (UTC) [reply]
Wouldn't that simply be growing meat in a lab. Remove all the unwanted parts - the stuff that isn't meat. You are left with meat. It isn't too hard to imagine. It also means that no turkeys would be killed to produce Thanksgiving dinners. I know - that is purely evil GM thinking. Thanksgiving without killing turkeys - I dare to imagine what kind of freak holiday that would be. -- kainaw 23:10, 26 November 2009 (UTC)[reply]
The Space Merchants (1952) features 'Chicken Little' which is exactly what Kainaw suggests (but the article is currently under maintenance, and presumably for that reason doesn't mention Chicken Little at present). --ColinFine (talk) 00:02, 27 November 2009 (UTC)[reply]

hoax? Bulgarian scientist claims alien contact

So what is this all about? There is a scientist called Lachezar Georgiev Filipov, who has a CV here, hosted by the Space Research Institute of Bulgaria. Today he is in the news for claiming that "aliens are already among us". Is this Bulgarian National Tease the Rest of the World Day? Or is it a reverse practical joke, designed to extract more funding -- "see, the public will believe anything, so we need mass education"? Or has his name been taken in vain? Or could it be that he actually believes this? BrainyBabe (talk) 16:22, 26 November 2009 (UTC)[reply]

How very interesting, I wonder if we should have messages relayed to us via 'the power of thought' as another one of the WP:Reliable sources Wikipedia uses? ;-) Dmcq (talk) 16:37, 26 November 2009 (UTC)[reply]
Is it wrong that I laughed at
"PROFESIONAL TRAINING
[...]
1981 Language qualification - England
in his CV? 86.144.145.238 (talk) 16:41, 26 November 2009 (UTC)[reply]
Yes, I noticed that too. The website of the institute looks c. 1997 -- in fact, I thought it might be a fake, but if so, someone has gone to many many pages of trouble. BrainyBabe (talk) 16:45, 26 November 2009 (UTC)[reply]
Lots of reputable science figures end up promoting crackpot ideas. Initially starting with a scientific-like approach, these people usually devolve into disreputable science figures, by way of ignoring factual evidence and the scientific method. Take a look, for example, at Edgar Mitchell. His outlandish views have put him outside the main fray of NASA and at odds with most scientific thought. Nimur (talk) 16:55, 26 November 2009 (UTC)[reply]
A little tracing of sources shows that everyone is ultimately reporting something that appeared in Novinar, a Bulgarian newspaper. You can read Novinar's interview with Filipov in broken English using Google translate: here. The blurb at the beginning suggests there is an earlier (Monday?) Novinar article publishing his thoughts, which sparked all of this. I cannot find this earlier article online, but then I have no grip on Bulgarian or the Cyrillic script. 86.144.145.238 (talk) 17:06, 26 November 2009 (UTC)[reply]
It also seems possible that there's an active and malicious effort to mistranslate key words and construe the interview in a different way than it was originally intended. Nimur (talk) 17:08, 26 November 2009 (UTC)[reply]
In which case, should we rope in some Language Desks regulars to comment on the English reporting compared to what was said in Bulgarian? 86.140.174.66 (talk) 17:16, 26 November 2009 (UTC)[reply]
How could the translation be intentionally mistranslated or misconstrued? Google translate is a purely automated tool, so there's no people involved in the translation process that could be intentionally malicious in the process.
Having read the automated English translation of the Bulgarian article, I see no evidence that would suggest that this is a hoax, or a joke, or that he doesn't genuinely believe it. He's just a scientist who wound up getting caught up in some bullshit. I think scientists as a whole are much less likely than the general public to get lured into believing nonsense like this, but it certainly does happen. Scientists are still just human. Red Act (talk) 17:59, 26 November 2009 (UTC)[reply]
Well, he has carefully examined 130 crop circles and asked the Vatican about it - you can only take 'due diligence' so far.
But to address the OP's question: I don't think we can accurately research his motives behind this announcement. If it's a hoax or a joke - it's a pretty pathetic one - maybe it was funnier in the original Bulgarian? He's claiming the messages are telepathic - and hearing voices in ones head (See Auditory hallucination) is a sign of schizophreniaormania. It's perfectly possible that this guy has a real medical problem. As an effort to extract more funding? Well, if that was the intent - I think it may have backfired - the linked article says that the finance minister is discussing it with the prime minister...I don't think that's going to end positively. SteveBaker (talk) 17:14, 26 November 2009 (UTC)[reply]
From the Bulgarian interview, I don't get that it's Filipov himself that thinks that he's getting telepathic messages through the crop circles, it's a woman by the name of Mariana Vezneva. Filipov has just become one of Vezneva's followers. Vezneva's web site says that the "Teachers" materialized twice in her home, so if she's having hallucinations, they're not just auditory ones. Red Act (talk) 20:38, 26 November 2009 (UTC)[reply]
"Mr Filipov said that even the seat of the Catholic church, the Vatican, had agreed that aliens existed." Indeed, they agree, but they call them God, seraphim, cherubim, etc. --Cookatoo.ergo.ZooM (talk) 18:09, 26 November 2009 (UTC)[reply]

How does the body sense temperature?

I don't mean, what does it do with the information, but the physical measurement used. I read Thermoregulation, Thermoception, and Homeostasis, and none of them mention it.

Temperature changes the speed of sound, the size of objects, the kinetic speed of gases, melting things, the speed of reactions, pressure in a container, electrical resistance. And yet none of those would seem to be measurable by the body. And accurately too - to less that .1 of a degree, and it's also tunable (fever) - how is it calibrated?

What physical thing does the body measure? I assume thermoregulation is tuned for accuracy, and thermoception for speed. Ariel. (talk) 18:53, 26 November 2009 (UTC)[reply]

To quote the Thermoception article, "The details of how temperature receptors work is still being investigated." :( If anyone does find refs for research on that, please add to the article. DMacks (talk) 21:00, 26 November 2009 (UTC)[reply]
There are specific neurons that sense heat -- they are cross-sensitive to capsaicin; there are also cold-sensing neurons, and these are cross-sensitive to menthol. That's why these and other items give us a "hot" or "cold" sensation. Perhaps what needs to be worked out and is still under investigation is how the body senses finer changes in temperature than basic hot + cold. It's possible, though, that the body just senses relative warmth of lack of warmth, just like the case of the three buckets (one with hot water, one with cold water, still a foot in each and after a minute, pour both contents into 3rd bucket, put both feet in and it feels hot for the foot that was in the cold and cold for the foot that was in the hot). DRosenbach (Talk | Contribs) 23:45, 26 November 2009 (UTC)[reply]
Even just basic hot/cold (without any fine measurements), and even simpler: simply measuring if something is hotter or colder than the body - how? What physical thing does it measure? And accurately measuring body temperature with no reference, or method calibration has to be even harder, and I wonder both how it does that, and what physical change does it measure? Ariel. (talk) 00:32, 27 November 2009 (UTC)[reply]
It really isn't known (and, given somatosensation is the last sensory system where very little is understood at the molecular level, when it is fully worked out it will probably earn the scientists a Nobel Prize). Scientists are largely focusing in Transient receptor potential ion channels, such as the TRPV and TRPA (channel)s, as the candidate proteins that detect temperature. I happen to have some colleagues that are at the forefront of this research. Frustratingly, the detection of actual temperature appears to be mechanistically different from the detection of chemicals that we perceive as having temperature (such as menthol or capsaicin), which complicates matters. But, without giving away too much unpublished data, they appear to have narrowed down the precise parts of these proteins that are temperature sensitive. They are now trying to work out what exactly happens to these specific amino-acids when you change their ambient temperature. That should provide a clue to the exact physical characteristic that precipitates the molecular change. Come back and ask this question in about 5 years, and I expect we will be able to answer it more fully. Rockpocket 01:55, 27 November 2009 (UTC)[reply]

harmful aspiration

Are societies which now are able to provide an abundance of food to everyone growing overweight due to the psychological effect of the aspiration to be wealthy and the fact that the wealthiest have become overweight? In other words if we aspired to break-even so to speak instead of aspiring to have wealth beyond our need would we find acquisition of normal weight to be the result of our ambition or the rule? 71.100.11.112 (talk) 20:36, 26 November 2009 (UTC) [reply]

Research has been published both indicating and refuting your assertion; e.g. Food insecurity is associated with increased risk of obesity (2003); and Obesity: the emerging cost of economic prosperity (2006). I searched Google Scholar for results with obesity and prosperity as my query. Nimur (talk) 22:57, 26 November 2009 (UTC)[reply]
I believe that the best explanation is simply that we evolved in a world without farming - where food was not abundant and where high-calorie foods were especially valuable to day-to-day survival. Hence, nothing in our biochemistry turns off the desire to eat or moderates our attraction to foods that are (in abundant quantities) harmful to us - but which were crucial to survival in early hominids. SteveBaker (talk) 23:27, 26 November 2009 (UTC)[reply]

Does blood really thicken in cold weather?

I have heard people say that their blood thickens in colder climates, and that this is why one can be more accustomed to the cold if one has lived their whole lives in, say, Canada versus visiting after spending their whole lives in Mexico.

Is this true? Or, is it just a joke, and something else causes one to become more accustomed to the cold? I suppose that it *could* be true, in that cold contracts things and heat expands it, but there are so many other things, such as nerve cells, that impact how we feel heat and cold.


Retrieved from "https://en.wikipedia.org/w/index.php?title=Wikipedia:Reference_desk/Science&oldid=328138580"

Categories: 
Science
Wikipedia resources for researchers
Hidden category: 
Non-talk pages that are automatically signed
 



This page was last edited on 27 November 2009, at 02:22 (UTC).

This version of the page has been revised. Besides normal editing, the reason for revision may have been that this version contains factual inaccuracies, vandalism, or material not compatible with the Creative Commons Attribution-ShareAlike License.



Privacy policy

About Wikipedia

Disclaimers

Contact Wikipedia

Code of Conduct

Developers

Statistics

Cookie statement

Mobile view



Wikimedia Foundation
Powered by MediaWiki